Antepartum 1

Réussis tes devoirs et examens dès maintenant avec Quizwiz!

In order to reassure and educate pregnant clients about changes in their cardiovascular system, maternity nurses should be aware that: a. A pregnant woman experiencing disturbed cardiac rhythm, such as sinus arrhythmia, requires close medical and obstetric observation no matter how healthy she otherwise may appear b. Changes in heart size and position and increases in blood volume create auditory changes from 20 weeks to term c. Palpitations are twice as likely to occur in twin gestations d. All of the above changes likely will occur

Changes in heart size and position and increases in blood volume create auditory changes from 20 weeks to term These auscultatory changes should be discernible after 20 weeks of gestation.

A nurse caring for a laboring woman should know that meconium is produced by: a. Fetal intestines b. Fetal kidneys c. Amniotic fluid d. The placenta

Fetal intestines As the fetus nears term, fetal waste products accumulate in the intestines as dark green to black, tarry meconium. Meconium is not produced by the fetal kidneys. Meconium should not be present in amniotic fluid. This may be an indication of fetal compromise. The placenta does not produce meconium.

A pregnant woman at 25 weeks of gestation tells the nurse that she dropped a pan last week and her baby jumped at the noise. Which response by the nurse is most accurate? a. That must have been a coincidence; babies cant respond like that. b. The fetus is demonstrating the aural reflex. c. Babies respond to sound starting at about 24 weeks of gestation. d. Let me know if it happens again; we need to report that to your midwife.

Babies respond to sound starting at about 24 weeks of gestation.

A 41-week pregnant multigravida presents in the labor and delivery unit after a nonstress test indicated that her fetus could be experiencing some difficulties in utero. Which diagnostic tool yields more detailed information about the fetus? a. Ultrasound for fetal anomalies b. Biophysical profile (BPP) c. Maternal serum alpha-fetoprotein (MSAFP) screening d. Percutaneous umbilical blood sampling (PUBS)

Biophysical profile (BPP) Real-time ultrasound permits detailed assessment of the physical and physiologic characteristics of the developing fetus and cataloging of normal and abnormal biophysical responses to stimuli. The BPP is a noninvasive, dynamic assessment of a fetus that is based on acute and chronic markers of fetal disease. An ultrasound for fetal anomalies would most likely have occurred earlier in the pregnancy. It is too late in the pregnancy to perform an MSAFP. Furthermore, it does not provide information related to fetal well-being. Indications for PUBS include prenatal diagnosis or inherited blood disorders, karyotyping of malformed fetuses, detection of fetal infection, determination of the acid-base status of the fetus with intrauterine growth restriction (IUGR), and assessment and treatment of isoimmunization and thrombocytopenia in the fetus.

While evaluating a client for osteoporosis, the nurse should be aware of what risk factor? a. African-American race b. Low protein intake c. Obesity d. Cigarette smoking

Cigarette smoking Smoking is associated with earlier and greater bone loss and decreased estrogen production. Women at risk for osteoporosis are likely to be Caucasian or Asian. Inadequate calcium intake is a risk factor for osteoporosis. Women at risk for osteoporosis are likely to be small boned and thin. Obese women have higher estrogen levels as a result of the conversion of androgens in the adipose tissue. Mechanical stress from extra weight also helps preserve bone mass.

The quadruple-marker test is used to assess the fetus for which condition? a. Down syndrome b. Diaphragmatic hernia c. Congenital cardiac abnormality d. Anencephaly

Down syndrome The maternal serum level of alpha-fetoprotein is used to screen for Down syndrome, neural tube defects, and other chromosomal anomalies.

A woman has a thick, white, lumpy, cottage cheeselike discharge, with patches on her labia and in her vagina. She complains of intense pruritus. The nurse practitioner orders which preparation for treatment? a. Fluconazole b. Tetracycline c. Clindamycin d. Acyclovir

Fluconazole Fluconazole, metronidazole, and clotrimazole are the drugs of choice to treat candidiasis. Tetracycline is used to treat syphilis. Clindamycin is used to treat bacterial vaginosis. Acyclovir is used to treat genital herpes.

For what reason is breastfeeding contraindicated? a. Hepatitis B b. Everted nipples c. History of breast cancer 3 years ago d. Human immunodeficiency virus (HIV) positive

Human immunodeficiency virus (HIV) positive Women who are HIV positive are discouraged from breastfeeding. Although hepatitis B antigen has not been shown to be transmitted through breast milk, as an added precaution infants born to HBsAg-positive women should receive the hepatitis B vaccine and immune globulin immediately after birth. Everted nipples are functional for breastfeeding. Newly diagnosed breast cancer is a contraindication to breastfeeding.

Of these psychosocial factors, which has the least negative effect on the health of the mother and/or fetus? a. Moderate coffee consumption b. Moderate alcohol consumption c. Cigarette smoke d. Emotional distress

Moderate coffee consumption Birth defects in humans have not been related to caffeine consumption. Pregnant women who consume more than 300 mg of caffeine daily may be at increased risk for miscarriage or intrauterine growth restriction (IUGR). Although the exact effects of alcohol in pregnancy have not been quantified, it exerts adverse effects on the fetus including fetal alcohol syndrome, fetal alcohol effects, learning disabilities, and hyperactivity. A strong, consistent, causal relation has been established between maternal smoking and reduced birth weight. Childbearing triggers profound and complex physiologic and psychologic changes. Evidence suggests a relationship between emotional distress and birth complications.

A woman is undergoing a nipple-stimulated contraction stress test (CST). She is having contractions that occur every 3 minutes. The fetal heart rate (FHR) has a baseline of approximately 120 beats/min without any decelerations. The interpretation of this test is said to be: a. Negative b. Positive c. Satisfactory d. Unsatisfactory

Negative Adequate uterine activity necessary for a CST consists of three contractions in a 10-minute time frame. If no decelerations are observed in the FHR pattern with the contractions, the findings are considered to be negative. A positive CST indicates the presence of repetitive later FHR decelerations. The term satisfactory is not applicable. The term unsatisfactory is not applicable.

A woman who is 16 weeks pregnant has come in for a follow-up visit with her significant other. In order to reassure the client regarding fetal well-being it is best for the nurse to: a. Assess the fetal heart tones with a Doppler stethoscope b. Measure the girth of the womans abdomen c. Complete an ultrasound examination (sonogram) d. Offer the woman and her family the opportunity to listen to the fetal heart tones

Offer the woman and her family the opportunity to listen to the fetal heart tones To provide the parents with the greatest sense of reassurance, the nurse should offer to have the client and her spouse the chance to listen to their babys heartbeat.

In the United States today: a. More than 20% of pregnancies meet the definition of high risk to either the mother or the infant b. Other than biophysical criteria, sociodemographic factors place both the mother and baby at risk c. High risk pregnancy status extends from first confirmation of pregnancy to birth d. High risk pregnancy is less critical a medical concern because of the reduction in family size and the decrease in unwanted pregnanciesOther than biophysical criteria, sociodemographic factors place both the mother and baby at risk

Other than biophysical criteria, sociodemographic factors place both the mother and baby at risk Sociodemographic risks include lack of prenatal care, low income, marital status, and ethnicity. Approximately 500,000 of the 4 million births (12.5%) in the United States are categorized as high risk. The high risk status for the mother extends through 30 days after childbirth. The reduction in family size and the decrease in unwanted pregnancies, along with technologic advances that facilitate pregnancies in previously infertile couples and advance the potential of care, have enhanced emphasis on delivering babies safely.

With regard to the assessment of female, male, and couple infertility, nurses should be aware that: a. The couples religious, cultural, and ethnic backgrounds provide emotional clutter that does not affect the clinical scientific diagnosis b. The investigation takes several months and can be very costly c. The woman is assessed first; if she is not the problem, the male partner is analyzed d. Semen analysis is for men; the postcoital test is for women

The investigation takes several months and can be very costly Fertility assessment and diagnosis take time, money, and commitment from the couple. Religious, cultural, and ethnic-bred attitudes about fertility and related issues always have an effect on diagnosis and assessment. Both partners are assessed systematically and simultaneously, as individuals and as a couple. Semen analysis is for men, but the postcoital test is for the couple.

In the first trimester, ultrasonography can be used to gain information on: a. Amniotic fluid volume b. The presence of maternal abnormalities c. Placental location and maturity d. Cervical length

The presence of maternal abnormalities Ultrasonography can detect certain uterine abnormalities such as bicornuate uterus, fibroids, and ovarian cysts. Amniotic fluid volume is not available via ultrasonography until the second or third trimester. Placental location and maturity are not available via ultrasonography until the second or third trimester.

In comparing the abdominal and transvaginal methods of ultrasound examination, nurses should explain to their clients that: a. Both require the woman to have a full bladder b. The abdominal examination is more useful in the first trimester c. Initially the transvaginal examination can be painful d. The transvaginal examination allows pelvic anatomy to be evaluated in greater detail

The transvaginal examination allows pelvic anatomy to be evaluated in greater detail The transvaginal examination also allows intrauterine pregnancies to be diagnosed earlier. The abdominal examination requires a full bladder; the transvaginal examination requires an empty one. The transvaginal examination is more useful in the first trimester; the abdominal examination works better after the first trimester. Neither method should be painful, although with the transvaginal examination the woman will feel pressure as the probe is moved.

A physician prescribes clomiphene citrate (Clomid, Serophene) for a woman experiencing infertility. She is very concerned about the risk of multiple births. The nurses most appropriate response is: a. This is a legitimate concern. Would you like to discuss this further before your treatment begins? b. No one has ever had more than triplets with Clomid. c. Ovulation will be monitored with ultrasound so that this will not happen. d. Ten percent is a very low risk, so you dont need to worry too much.

This is a legitimate concern. Would you like to discuss this further before your treatment begins? The incidence of multiple pregnancies with the use of these medications is higher than 25%. The clients concern is legitimate and should be discussed so that she can make an informed decision. Stating that no one has ever had more than triplets with Clomid is inaccurate and negates the clients concerns. Ultrasound cannot ensure that a multiple pregnancy will not occur. Ten percent is inaccurate. Furthermore, the clients concern is discredited with a statement such as dont worry.

A 39-year-old primigravida thinks that she is about 8 weeks pregnant, although she has had irregular menstrual periods all her life. She has a history of smoking approximately one pack of cigarettes a day, but she tells you that she is trying to cut down. Her laboratory data are within normal limits. What diagnostic technique could be used with this pregnant woman at this time? a. Ultrasound examination b. Maternal serum alpha-fetoprotein (MSAFP) screening c. Amniocentesis d. Nonstress test (NST)

Ultrasound examination An ultrasound examination could be done to confirm the pregnancy and determine the gestational age of the fetus. It is too early in the pregnancy to perform an MSAFP screening; it is performed at 16 to 18 weeks of gestation. An amniocentesis is performed if the MSAFP levels are abnormal or if fetal/maternal anomalies are detected. This procedure is not performed until 16 to 18 weeks of gestation. An NST is performed to assess fetal well-being in the third trimester.

Which symptom is considered a first-trimester warning sign and should be reported immediately by the pregnant woman to her health care provider? a. Nausea with occasional vomiting b. Fatigue c. Urinary frequency d. Vaginal bleeding

Vaginal bleeding Signs and symptoms that must be reported include severe vomiting, fever and chills, burning on urination, diarrhea, abdominal cramping, and vaginal bleeding. These symptoms may be signs of complications of the pregnancy.

With regard to a womans reordering of personal relationships during pregnancy, the maternity nurse should be aware that: a. Because of the special motherhood bond, a woman's relationship with her mother is even more important than with the father of the child b. Nurses need not get involved in any sexual issues the couple has during pregnancy, particularly if they have trouble communicating them to each other c. Women usually express two major relationship needs during pregnancy: feeling loved and valued and having the child accepted by the father d. The woman's sexual desire is likely to be highest in the first trimester because of the excitement and because intercourse is physically easier

Women usually express two major relationship needs during pregnancy: feeling loved and valued and having the child accepted by the father Love and support help a woman feel better about her pregnancy. The most important person to the pregnant woman is usually the father. Nurses can facilitate communication between partners about sexual matters if, as is common, they are nervous about expressing their worries and feelings. The second trimester is the time when a womans sense of well-being, along with certain physical changes, increases her desire for sex. Desire is down in the first and third trimesters.

A client asks her nurse, My doctor told me that he is concerned with the grade of my placenta because I am overdue. What does that mean? The best response by the nurse is: a. Your placenta changes as your pregnancy progresses, and it is given a score that indicates the amount of calcium deposits it has. The more calcium deposits, the higher the grade, or number, that is assigned to the placenta. It also means that less blood and oxygen can be delivered to your baby. b. Your placenta isnt working properly, and your baby is in danger. c. This means that we will need to perform an amniocentesis to detect if you have any placental damage. d. Dont worry about it. Everything is fine.

Your placenta changes as your pregnancy progresses, and it is given a score that indicates the amount of calcium deposits it has. The more calcium deposits, the higher the grade, or number, that is assigned to the placenta. It also means that less blood and oxygen can be delivered to your baby. Calcium deposits are of significance in postterm pregnancies. Ultrasonography can be used to determine placental aging. Although stating that the clients placenta isnt working properly and the baby is in danger may be a valid response, it does not reflect therapeutic communication techniques and is likely to alarm the client. An ultrasound, not an amniocentesis, is the method of assessment used to determine placental maturation. Telling the client not to worry is not appropriate and discredits her concerns.

A womans obstetric history indicates that she is pregnant for the fourth time, and all her children from previous pregnancies are living. One was born at 39 weeks of gestation, twins were born at 34 weeks of gestation, and another child was born at 35 weeks of gestation. What is her gravidity and parity using the GTPAL system? a. 3-1-1-1-3 b. 4-1-2-0-4 c. 3-0-3-0-3 d. 4-2-1-0-3

4-1-2-0-4 4-1-2-0-4 is the correct calculation of this womans gravidity and parity. The numbers reflect the womans gravidity and parity information. Using the GPTAL system, her information is calculated as: G reflects the total number of times the woman has been pregnant; she is pregnant for the fourth time. T indicates the number of pregnancies carried to term, not the number of deliveries at term; only one of her pregnancies has resulted in a fetus at term. P is the number of pregnancies that resulted in a preterm birth; the woman has had two pregnancies in which she delivered preterm. A signifies whether the woman has had any abortions or miscarriages before the period of viability; she has not. L signifies the number of children born that currently are living; the woman has four children.

Human chorionic gonadotropin (hCG) is an important biochemical marker for pregnancy and therefore the basis for many tests. A maternity nurse should be aware that: a. hCG can be detected as early as 2 weeks after conception b. The hCG level increases gradually and uniformly throughout pregnancy c. Much lower than normal increases in the level of hCG may indicate a postdate pregnancy d. A higher than normal level of hCG may indicate an ectopic pregnancy or Down syndrome

A higher than normal level of hCG may indicate an ectopic pregnancy or Down syndrome Higher levels also could be a sign of multiple gestation. hCG can be detected as early as 7 to 10 days after conception. The hCG level fluctuates during pregnancy, peaking, declining, stabilizing, and then increasing again. Abnormally slow increases may indicate impending miscarriage.

Which statement about pregnancy is accurate? a. A normal pregnancy lasts about 10 lunar months b. A trimester is one third of a year c. The prenatal period extends from fertilization to conception d. The estimated date of confinement (EDC) is how long the mother will have to be bedridden after birth

A normal pregnancy lasts about 10 lunar months A lunar month lasts 28 days, or 4 weeks. Pregnancy spans 9 calendar months but 10 lunar months. A trimester is on

Probable signs of pregnancy are: a. Determined by ultrasound b. Observed by the health care provider c. Reported by the client d. Diagnostic tests

Observed by the health care provider Probable signs are those detected through trained examination.

The mucous plug that forms in the endocervical canal is called the: a. Operculum b. Leukorrhea c. Funic souffle d. Ballottement

Operculum The operculum protects against bacterial invasion.

A nurse caring for a pregnant client must understand that the hormone essential for maintaining pregnancy is: a. Estrogen b. Human chorionic gonadotropin (hCG) c. Oxytocin d. Progesterone

Progesterone Progesterone is essential for maintaining pregnancy; it does so by relaxing smooth muscles. This reduces uterine activity and prevents miscarriage.

A woman has just moved to the United States from Mexico. She is 3 months pregnant and has arrived for her first prenatal visit. During her assessment interview you discover that she has not had any immunizations. Which immunizations should she receive at this point in her pregnancy? Choose all that apply. a. Tetanus b. Diphtheria c. Chickenpox d. Rubella e. Hepatitis B

Tetanus Diphtheria Hepatitis B Vaccines consisting of killed viruses may be used. Those that may be administered during pregnancy include tetanus, diphtheria, recombinant hepatitis B, and rabies vaccines.

The various systems and organs develop at different stages. Which statement is accurate? a. The cardiovascular system is the first organ system to function in the developing human. b. Hematopoiesis originating in the yolk sac begins in the liver at 10 weeks. c. The body changes from straight to C-shaped at 8 weeks. d. The gastrointestinal system is mature at 32 weeks.

The cardiovascular system is the first organ system to function in the developing human. The heart is developmentally complete by the end of the embryonic stage. Hematopoiesis begins in the liver during the sixth week. The body becomes C-shaped at 21 weeks. The gastrointestinal system is complete at 36 weeks.

A postmenopausal woman who is 54 years old has been diagnosed with two leiomyomas. What assessment finding is most commonly associated with the presence of leiomyomas? a. Abnormal uterine bleeding b. Diarrhea c. Weight loss d. Acute abdominal pain

Abnormal uterine bleeding Most women are asymptomatic. Abnormal uterine bleeding is the most common symptom of leiomyomas (fibroids). Diarrhea is not commonly associated with leiomyomas (fibroids). Weight loss does not usually occur in the woman with leiomyomas (fibroids). The client with leiomyomas (fibroids) is unlikely to experience abdominal pain.

What is not a potential pitfall or disadvantage of the fertility awareness methods (FAMs)? a. Adherence to strict record keeping b. Alterations in the menstrual flow pattern with chemicals and hormones c. Decreased effectiveness in women with irregular cycles d. Time-consuming training sessions

Alterations in the menstrual flow pattern with chemicals and hormones The absence of chemicals or hormones to alter the menstrual flow is an advantage. The strict record keeping FAMs require creates a potential risk. FAMs are less effective for women with irregular cycles. FAMs require time-consuming training for effective use.

The absence or cessation of menstrual flow is known as: a. Amenorrhea b. Dysmenorrhea c. Menorrhagia d. Metrorrhagia

Amenorrhea Amenorrhea is most often the result of pregnancy, although it could be a sign of conditions that require treatment. Dysmenorrhea is painful menstruation that begins 2 to 6 months after menarche. Menorrhagia is abnormally profuse or excessive bleeding from the uterus. Metrorrhagia is bleeding between periods. It can be caused by progestin injections and implants.

With regard to maternal, fetal, and neonatal health problems, nurses should be aware that: a. Infection has replaced pulmonary embolism as one of the three top causes of maternal death attributable to pregnancy b. The leading cause of death in the neonatal period is disorders related to short gestation and low birth weight c. Factors related to the maternal death rate include age and marital status but not race d. Antepartum fetal deaths can best be prevented by better recognizing and responding to abnormalities of pregnancy and labor

Antepartum fetal deaths can best be prevented by better recognizing and responding to abnormalities of pregnancy and labor Medical teams need to be alert to signs of trouble. Race is a factor. African-American maternal mortality rates are more than three times higher than those for Caucasian women. Infection used to be an important cause of maternal death; it has been replaced by pulmonary embolism. The leading cause of death in the neonatal period is congenital anomalies. Race is a factor. African-American maternal mortality rates are more than three times higher than those for Caucasian women.

Which contraceptive method best protects against sexually transmitted infections (STIs) and human immunodeficiency virus (HIV)? a. Periodic abstinence b. Barrier methods c. Hormonal methods d. They all offer about the same protection.

Barrier methods Barrier methods, such as condoms, best protect against STIs and HIV. Periodic abstinence offers no protection against STIs or HIV. Hormonal methods, such as birth control pills, offer no protection against STIs or HIV. Periodic abstinence and hormonal methods (the pill) offer no protection against STIs or HIV.

When evaluating a client for sexually transmitted infections (STIs), the nurse should be aware that the most common bacterial STI is: a. Gonorrhea b. Syphilis c. Chlamydia d. Candidiasis

Chlamydia Chlamydia is the most common and fastest spreading STI among American women, with an estimated 3 million new cases each year. Gonorrhea is a bacterial STI; however, it is not the most common among American women. Syphilis is a bacterial STI; however, it is not the most common among American women. Candidiasis is caused by a fungus, not by bacteria.

A pregnant woman tells her nurse that she is worried about the blotchy, brownish coloring over her cheeks, nose, and forehead. The nurse can reassure her that this is a normal condition related to hormonal change, commonly called the mask of pregnancy or, scientifically: a. Chloasma b. Linea nigra c. Striae gravidarum d. Palmar erythema

Chloasma Chloasma, the mask of pregnancy, usually fades after birth.

The _____ is/are responsible for oxygen and carbon dioxide transport to and from the maternal bloodstream. a. Decidua basalis b. Blastocyst c. Germ layer d. Chorionic villi

Chorionic villi Chorionic villi are finger-like projections that develop out of the trophoblast and extend into the blood-filled spaces of the endometrium. The villi obtain oxygen and nutrients from the maternal bloodstream and dispose of carbon dioxide and waste products into the maternal blood. The decidua basalis is the portion of the decidua (endometrium) under the blastocyst where the villi attach. The blastocyst is the embryonic development stage after the morula. Implantation occurs at this stage. The germ layer is a layer of the blastocyst.

The nurse should be aware that the pinch test is used to: a. Check the sensitivity of the nipples b. Determine whether the nipple is everted or inverted c. Calculate the adipose buildup in the abdomen d. See whether the fetus has become inactive

Determine whether the nipple is everted or inverted The pinch test is used to determine whether the nipple is everted or inverted. Nipples must be everted to allow breastfeeding.

Appendicitis may be difficult to diagnose in pregnancy because the appendix is: a. Displaced upward and laterally, high and to the right b. Displaced upward and laterally, high and to the left c. Deep at McBurneys point d. Displaced downward and laterally, low and to the right

Displaced upward and laterally, high and to the right

In their role of implementing a plan of care for infertile couples, nurses should: a. Be comfortable with their own sexuality and nonjudgmental about others to effectively counsel their clients b. Know about such nonmedical remedies as diet, exercise, and stress management c. Be able to direct clients to sources of information about what herbs to take that might help and which ones to avoid d. Do all of these plus be knowledgeable about potential drug and surgical remedies

Do all of these plus be knowledgeable about potential drug and surgical remedies Nurses should also be cognizant that a number of surgical procedures can be used for problems related to female infertility. Nurses can help couples express and discuss their feelings as honestly as possible. The nurse must be comfortable with his or her own sexuality. The nurse should be aware that simple changes in lifestyle may be effective in assisting the infertile couple. Most herbal remedies have not been proven effective.

A nurse sees a woman for the first time when she is 30 weeks pregnant. The woman has smoked throughout the pregnancy, and fundal height measurements now are suggestive of intrauterine growth restriction (IUGR) in the fetus. In addition to ultrasound to measure fetal size, what is another tool useful in confirming the diagnosis? a. Doppler blood flow analysis b. Contraction stress test (CST) c. Amniocentesis d. Daily fetal movement counts

Doppler blood flow analysis Doppler blood flow analysis allows the examiner to study the blood flow noninvasively in the fetus and the placenta. It is a helpful tool in the management of high risk pregnancies because of IUGR, diabetes mellitus, multiple fetuses, or preterm labor. Because of the potential risk of inducing labor and causing fetal distress, a CST is not performed on a woman whose fetus is preterm. Indications for an amniocentesis include diagnosis of genetic disorders or congenital anomalies, assessment of pulmonary maturity, and the diagnosis of fetal hemolytic disease, not IUGR. Fetal kick count monitoring is performed to monitor the fetus in pregnancies complicated by conditions that may affect fetal oxygenation. Although this may be a useful tool at some point later in this womans pregnancy, it is not used to diagnose IUGR.

The most common perinatal complications associated with bacterial sexually transmitted infections (STIs) are: a. Preterm labor and preterm birth b. Newborn eye infections and low Apgar scores c. Nausea, vomiting, and frequent urinary tract infections d. Congenital anomalies and infertility

Preterm labor and preterm birth Risk factors associated with bacterial STIs include preterm labor and preterm birth, miscarriage, and intrauterine growth restriction (IUGR). Perinatal complications of bacterial gonorrhea may lead to newborn eye infections but not necessarily to low Apgar scores. Nausea, vomiting, and frequent urinary tract infections are not associated with bacterial STIs. TORCH infections (which are not entirely sexually transmitted) are associated with congenital anomalies. Untreated STIs may progress to pelvic inflammatory disease (PID) and infertility.

From the age of menarche to menopause, women remain at risk for structural disorders and neoplasms of the reproductive system. When caring for these clients the nurse must begin by assessing the womans knowledge of the disorder, its management, and prognosis. This assessment should be followed by a nursing diagnosis. Which diagnosis does not address the psychologic effect of these disorders? a. Anxiety related to surgical procedures b. Disturbed body image as a result of changes in anatomy c. Risk for injury related to lack of skill for self-care d. Interrupted family processes

Risk for injury related to lack of skill for self-care Although risk for injury related to lack of skill for self-care is appropriate to the clients condition, it is more suited to the clients learning needs than the psychologic effect. Anxiety related to surgical procedures is appropriate for addressing psychosocial concerns. The client may also develop anxiety related to the diagnosis and prognosis, whether or not surgery is required. Disturbed body image is an applicable diagnosis. Changes in anatomy and function may also result in low self-esteem and ineffective coping skills. Interrupted family processes is a possible acceptable diagnosis. Functional and anatomic changes may result in the clients inability to fulfill her familial role. Depending on the severity of her condition, it could also lead to social isolation.

The musculoskeletal system adapts to the changes that occur during pregnancy. A woman can expect to experience what change? a. Her center of gravity will shift backward. b. She will have increased lordosis. c. She will have increased abdominal muscle tone. d. She will notice decreased mobility of her pelvic joints.

She will have increased lordosis. An increase in the normal lumbosacral curve (lordosis) develops, and a compensatory curvature in the cervicodorsal region develops to help her maintain her balance. The center of gravity shifts forward. She will have decreased muscle tone. She will notice increased mobility of her pelvic joints.

A man smokes two packs of cigarettes a day. He wants to know if smoking is contributing to the difficulty he and his wife are having getting pregnant. The nurses most appropriate response is: a. Your sperm count seems to be okay in the first semen analysis. b. Only marijuana cigarettes affect sperm count. c. Smoking can give you lung cancer, even though it has no effect on sperm. d. Smoking can reduce the quality of your sperm.

Smoking can reduce the quality of your sperm. Cigarette smoking has been associated with abnormal sperm, a decreased number of sperm and chromosome damage. Sperm counts vary from day to day and are dependent on emotional and physical status and sexual activity. Therefore, a single analysis may be inconclusive. A minimum of two analyses must be performed several weeks apart to assess male fertility. Marijuana use may depress the number and motility of sperm. Smoking is indeed a causative agent for lung cancer. The use of tobacco also has detrimental effects on sperm.

Which time-based description of a stage of development in pregnancy is accurate? a. Viability22 to 37 weeks since the last menstrual period (assuming a fetal weight greater than 500 g) b. Termpregnancy from the beginning of week 38 of gestation to the end of week 42 c. Pretermpregnancy from 20 to 28 weeks d. Postdatepregnancy that extends beyond 38 weeks

Term pregnancy from the beginning of week 38 of gestation to the end of week 42 Term is 38 to 42 weeks of gestation. Viability is the ability of the fetus to live outside the uterus before coming to term, or 22 to 24 weeks since the last menstrual period. Preterm is 20 to 37 weeks of gestation. Postdate or postterm is a pregnancy that extends beyond 42 weeks or what is considered the limit of full term.

A woman currently uses a diaphragm and spermicide for contraception. She asks the nurse what the major differences are between the cervical cap and diaphragm. The nurses most appropriate response is: a. No spermicide is used with the cervical cap, so its less messy. b. The diaphragm can be left in place longer after intercourse. c. Repeated intercourse with the diaphragm is more convenient. d. The cervical cap can safely be used for repeated acts of intercourse without adding more spermicide later.

The cervical cap can safely be used for repeated acts of intercourse without adding more spermicide later. The cervical cap can be inserted hours before sexual intercourse without the need for additional spermicide later. No additional spermicide is required for repeated acts of intercourse. Spermicide should be used inside the cap as an additional chemical barrier. The cervical cap should remain in place for 6 hours after the last act of intercourse. Repeated intercourse with the cervical cap is more convenient because no additional spermicide is needed.

A 3-year-old girls mother is 6 months pregnant. What concern is this child likely to verbalize? a. How the baby will get out b. What the baby will eat c. Whether her mother will die d. What color eyes the baby has

What the baby will eat By age 3 or 4, children like to be told the story of their own beginning and accept it being compared with the present pregnancy. They like to listen to the fetal heartbeat and feel the baby move. Sometimes they worry about how the baby is being fed and what it wears.

A woman who has a seizure disorder and takes barbiturates and phenytoin sodium daily asks the nurse about the pill as a contraceptive choice. The nurses most appropriate response would be: a. This is a highly effective method, but it has some side effects. b. Your current medications will reduce the effectiveness of the pill. c. The pill will reduce the effectiveness of your seizure medication. d. This is a good choice for a woman of your age and personal history.

Your current medications will reduce the effectiveness of the pill. Because the liver metabolizes oral contraceptives, their effectiveness is reduced when they are taken simultaneously with anticonvulsants. Stating that the pill is an effective birth control method with side effects is a true statement, but it is not the most appropriate response. The anticonvulsant reduces the effectiveness of the pill, not the other way around. Stating that the pill is a good choice for a woman of her age and personal history does not teach the client that the effectiveness of the pill may be reduced because of her anticonvulsant therapy.

A nurse providing care to a pregnant woman should know that all are normal gastrointestinal changes in pregnancy except: a. Ptyalism b. Pyrosis c. Pica d. Decreased peristalsis

Pica (a desire to eat nonfood substances) is an indication of iron deficiency and should be evaluated.

A newly married couple plans to use natural family planning. It is important for them to know how long an ovum can live after ovulation. The nurse knows that teaching is effective when the couple responds that an ovum is considered fertile for: a. 6 to 8 hours b. 24 hours c. 2 to 3 days d. 1 week

24 hours Ova are considered fertile for about 24 hours after ovulation. Ova are considered fertile for much longer than 6 to 8 hours. Most remain fertile for 24 hours. Ova do not remain fertile for 2 to 3 days. If unfertilized by a sperm, the ovum degenerates and is reabsorbed. Ova do not remain viable for 1 week. After 24 hours the ovum degenerates and is reabsorbed.

At approximately _____ weeks of gestation, lecithin is forming on the alveolar surfaces, the eyelids open, and the fetus measures approximately 27 cm crown to rump and weighs approximately 1110 g. a. 20 b. 24 c. 28 d. 30

28 These milestones occur at 28 weeks. These milestones are not completed by 20 weeks of gestation. These milestones in human development are not completed at 24 weeks of gestation. These specific milestones are reached as early as 28, not 30, weeks of gestation.

Prenatal testing for the human immunodeficiency virus (HIV) is recommended for which women? a. All women, regardless of risk factors b. A woman who has had more than one sexual partner c. A woman who has had a sexually transmitted infection d. A woman who is monogamous with her partner

All women, regardless of risk factors An HIV test is recommended for all women, regardless of risk factors. The incidence of perinatal transmission from an HIV-positive mother to her fetus ranges from 25% to 35%. Women who test positive for HIV can then be treated.

During the first trimester a woman can expect which of the following changes in her sexual desire? a. An increase, because of enlarging breasts b. A decrease, because of nausea and fatigue c. No change d. An increase, because of increased levels of female hormones

A decrease, because of nausea and fatigue Maternal physiologic changes, such as breast enlargement, nausea, fatigue, abdominal changes, perineal enlargement, leukorrhea, pelvic vasocongestion, and orgasmic responses, may affect sexuality and sexual expression. Libido may be depressed in the first trimester but often increases during the second and third trimesters.

With regard to amniocentesis, nurses should be aware that: a. Because of new imaging techniques, it is now possible in the first trimester b. Despite the use of ultrasonography, complications still occur in the mother or infant in 5% to 10% of cases c. Administration of RhoD immunoglobulin may be necessary d. The presence of meconium in the amniotic fluid is always cause for concern

Administration of RhoD immunoglobulin may be necessary Due to the possibility of fetomaternal hemorrhage, administration of RhoD immunoglobulin is the standard of practice after amniocentesis for women who are Rh negative. Amniocentesis is possible after the fourteenth week of pregnancy when the uterus becomes an abdominal organ. Complications occur in less than 1% of cases; many have been minimized or eliminated through the use of ultrasonography. Meconium in the amniotic fluid before the beginning of labor is not usually a problem.

A nurse caring for a newly pregnant woman advises her that ideally prenatal care should begin: a. Before the first missed menstrual period b. After the first missed menstrual period c. After the second missed menstrual period d. After the third missed menstrual period

After the first missed menstrual period Prenatal care ideally should begin soon after the first missed menstrual period. This offers the greatest opportunities to ensure the health of the expectant mother and her infant.

A woman at 10 weeks of gestation who is seen in the prenatal clinic with presumptive signs and symptoms of pregnancy likely has: a. Amenorrhea b. Positive pregnancy test c. Chadwick sign d. Hegar sign

Amenorrhea Amenorrhea is a presumptive sign of pregnancy. Presumptive signs of pregnancy are those felt by the woman. A positive pregnancy test is a probable sign of pregnancy. The presence of the Chadwick sign is a probable sign of pregnancy. The presence of the Hegar sign is a probable sign of pregnancy.

At 35 weeks of pregnancy, a woman experiences preterm labor. Although tocolytics are administered and she is placed on bed rest, she continues to experience regular uterine contractions and her cervix is beginning to dilate and efface. What is an important test for fetal well-being at this time? a. Percutaneous umbilical blood sampling (PUBS) b. Ultrasound for fetal size c. Amniocentesis for fetal lung maturity d. Nonstress test (NST)

Amniocentesis for fetal lung maturity Amniocentesis is performed to assess fetal lung maturity in the event of a preterm birth. Indications for PUBS include prenatal diagnosis or inherited blood disorders, karyotyping of malformed fetuses, detection of fetal infection, determination of the acid-base status of the fetus with intrauterine growth restriction (IUGR), and assessment and treatment of isoimmunization and thrombocytopenia in the fetus. Determination of fetal size by ultrasound typically is done during the second trimester and is not indicated in this scenario. An NST measures the fetal response to fetal movement in a noncontracting mother.

Some pregnant clients may complain of changes in their voice and impaired hearing. The nurse can tell these clients that these are common reactions to: a. A decreased estrogen level b. Displacement of the diaphragm, resulting in thoracic breathing c. Congestion and swelling, which occur because the upper respiratory tract has become more vascular d. Increased blood volume

Congestion and swelling, which occur because the upper respiratory tract has become more vascular Estrogen levels increase, causing the upper respiratory tract to become more vascular; this produces swelling and congestion in the nose and ears and therefore voice changes and impaired hearing.

A pregnant woman at 18 weeks of gestation calls the clinic to report that she has been experiencing occasional backaches of mild to moderate intensity. The nurse recommends that she: a. Do Kegel exercises b. Do pelvic rock exercises c. Use a softer mattress d. Stay in bed for 24 hours

Do pelvic rock exercises Pelvic rock exercises may help stretch and strengthen the abdominal and lower back muscles and relieve low back pain. Kegel exercises increase the tone of the pelvic area, not the back. A softer mattress may not provide the support needed to maintain proper alignment of the spine and may contribute to back pain. Stretching and other exercises to relieve back pain should be performed several times a day.

In response to requests by the U.S. Public Health Service for new models of prenatal care, an innovative new approach to prenatal care known as centering pregnancy was developed. Which statement accurately applies to the centering model of care? a. Group sessions begin with the first prenatal visit. b. At each visit blood pressure, weight, and urine dipsticks are obtained by the nurse. c. Eight to 12 women are placed in gestational-age cohort groups. d. Outcomes are similar to traditional prenatal care.

Eight to 12 women are placed in gestational-age cohort groups. Gestational-age cohorts comprise the groups, with approximately 8 to 12 women in each group. This group remains intact throughout the pregnancy. Individual follow-up visits are scheduled as needed.

The phenomenon of someone other than the mother-to-be experiencing pregnancy-like symptoms such as nausea and weight gain applies to the: a. Mother of the pregnant woman b. Couples teenage daughter c. Sister of the pregnant woman d. Expectant father

Expectant father An expectant fathers experiencing of his partners pregnancy-like symptoms is called the couvade syndrome. The mother of the pregnant woman is unlikely to experience this phenomenon. She may be excited about becoming a grandmother or see this as a reminder that she is getting old. A couples teenage daughter is usually preoccupied with her own sexual development and may have difficulty accepting the overwhelming evidence of her parents sexual activity. It is the father of the pregnant woman, not the sister, who experiences these symptoms.

A woman arrives at the clinic seeking confirmation that she is pregnant. The following information is obtained: She is 24 years old with a body mass index (BMI) of 17.5. She admits to having used cocaine several times during the past year and drinks alcohol occasionally. Her blood pressure is 108/70 mm Hg, her pulse rate is 72 beats/min, and her respiratory rate is 16 breaths/min. The family history is positive for diabetes mellitus and cancer. Her sister recently gave birth to an infant with a neural tube defect (NTD). Which characteristics place the woman in a high risk category? a. Blood pressure, age, BMI b. Drug/alcohol use, age, family history c. Family history, blood pressure (BP), BMI d. Family history, BMI, drug/alcohol abuse

Family history, BMI, drug/alcohol abuse Her family history of NTD, low BMI, and substance abuse are high risk factors of pregnancy. The womans BP is normal, and her age does not put her at risk. Her BMI is low and may indicate poor nutritional status, which is a high risk. The womans drug/alcohol (ETOH) use and family history put her in a high risk category, but her age does not. The womans family history puts her in a high risk category. Her BMI is low and may indicate poor nutritional status, which is high risk. Her BP is normal.

Intrauterine growth restriction (IUGR) is associated with what pregnancy-related risk factors? Choose all that apply. a. Poor nutrition b. Maternal collagen disease c. Gestational hypertension d. Premature rupture of membranes e. Smoking

Poor nutrition Maternal collagen disease Gestational hypertension Smoking Poor nutrition, maternal collagen disease, gestational hypertension, and smoking are risk factors associated with the occurrence of IUGR.

Which statement concerning neurologic and sensory development is accurate? a. Brain waves have been recorded on an electroencephalogram as early as the end of the first trimester (12 weeks). b. Fetuses respond to sound by 24 weeks and can be soothed by the sound of the mothers voice. c. Eyes are first receptive to light at 34 to 36 weeks. d. At term, the fetal brain is at least one third the size of an adult brain.

Fetuses respond to sound by 24 weeks and can be soothed by the sound of the mothers voice. Hearing develops early and is fully developed at birth. Brain waves have been recorded at week 8. Eyes are receptive to light at 28 weeks. The fetal brain is about one fourth the size of an adult brain.

In comparison to contraction stress tests (CSTs), the nonstress test (NST) for antepartum fetal assessment: a. Has no known contraindications b. Has fewer false-positive results c. Is more sensitive in detecting fetal compromise d. Is slightly more expensive

Has no known contraindications The CST has several contraindications. The NST has a high rate of false-positive results. The NST is less sensitive than the CST. The NST is relatively inexpensive.

During labor, a doula is expected to: a. Help the woman do Lamaze breathing techniques and provide support to the woman and her partner b. Check the fetal monitor tracing for effects of the labor process on the fetal heart rate c. Take the place of the father as a coach and support provider d. Administer pain medications as needed by the woman

Help the woman do Lamaze breathing techniques and provide support to the woman and her partner A doula is professionally trained to provide labor support, including physical, emotional, and informational, to women and their partners during labor and birth.

A woman is 3 months pregnant. At her prenatal visit she tells the nurse that she doesn't know what is happening; one minute she is happy that she is pregnant and the next minute she cries for no reason. Which response by the nurse is most appropriate? a. Don't worry about it; you'll feel better in a month or so. b. Have you talked to your husband about how you feel? c. Perhaps you really don't want to be pregnant. d. Hormone changes during pregnancy commonly result in mood swings.

Hormone changes during pregnancy commonly result in mood swings. Explaining that hormone changes can result in mood swings is an accurate statement and the most appropriate response by the nurse.

What represents a typical progression through the phases of a woman's establishing a relationship with the fetus? a. Accepts the fetus as distinct from herself accepts the biologic fact of pregnancy has a feeling of caring and responsibility b. Fantasizes about the child's gender and personality views the child as part of herself becomes introspective c. Views the child as part of herself has feelings of well-being accepts the biologic fact of pregnancy d. I am pregnant I am going to have a babyI am going to be a mother.

I am pregnant I am going to have a baby I am going to be a mother. The woman first centers on herself as pregnant, then on the baby as an entity separate from herself, and then on her responsibilities as a mother. The expressions I am pregnant, I am going to have a baby, and I am going to be a mother sum up the progression through the three phases.

The diagnosis of pregnancy is based on which positive signs of pregnancy? Choose all that apply. a. Identification of fetal heartbeat b. Palpation of fetal outline c. Visualization of the fetus d. verification of fetal movement e. Positive human chorionic gonadotropin (hCG) test

Identification of fetal heartbeat Visualization of the fetus verification of fetal movement Identification of fetal heartbeat, visualization of the fetus, and verification of fetal movement are all positive, objective signs of pregnancy.

Dental care during pregnancy is an important component of good prenatal care. Maternity nurses reinforce this by providing which of the following instructions to their clients: a. Regular brushing and flossing may not be necessary during early pregnancy because it may stimulate the woman who is already nauseated to vomit. A cleaning is all that is necessary b. Dental surgery, in particular, is contraindicated during pregnancy and should be delayed until after delivery c. If dental treatment is necessary, the woman will be most comfortable with it in the second trimester d. If a woman has dental anxiety, dental care may interfere with the expectant mothers need to practice conscious relaxation and preparation for labor

If dental treatment is necessary, the woman will be most comfortable with it in the second trimester The second trimester is best for dental treatment because that is when the woman will be able to sit most comfortably in the dental chair.

In order to reassure and educate pregnant clients about the functioning of their kidneys in eliminating waste products, maternity nurses should be aware that: a. Increased urinary output makes pregnant women less susceptible to urinary infection b. Increased bladder sensitivity and then compression of the bladder by the enlarging uterus result in the urge to urinate even if the bladder is almost empty c. Renal (kidney) function is more efficient when the woman assumes a supine position d. Using diuretics during pregnancy can help keep kidney function regular

Increased bladder sensitivity and then compression of the bladder by the enlarging uterus result in the urge to urinate even if the bladder is almost empty First bladder sensitivity and then compression of the bladder by the uterus result in the urge to urinate more often.

In order to reassure and educate pregnant clients about changes in the cervix, vagina, and position of the fetus, nurses should be aware that: a. Because of a number of changes in the cervix, abnormal Papanicolaou (Pap) tests are much easier to evaluate b. Quickening is a technique of palpating the fetus to engage it in passive movement c. The deepening color of the vaginal mucosa and cervix (Chadwick sign) usually appears in the second trimester or later as the vagina prepares to stretch during labor d. Increased vascularity of the vagina increases sensitivity and may lead to a high degree of arousal, especially in the second trimester

Increased vascularity of the vagina increases sensitivity and may lead to a high degree of arousal, especially in the second trimester Increased sensitivity and an increased interest in sex sometimes go together. This frequently occurs during the second trimester.

Congenital disorders refer to those conditions that are present at birth. These disorders may be inherited and caused by environmental factors or maternal malnutrition. Toxic exposures have the greatest effect on development between 15 and 60 days of gestation. For the nurse to be able to conduct a complete assessment of the newly pregnant client, he or she should be knowledgeable regarding known human teratogens, which include: a. Infections b. Radiation c. Maternal conditions d. Drugs e. Chemicals

Infections Radiation Maternal conditions Drugs Chemicals Exposure to radiation and a number of infections may result in profound congenital deformities. These include but are not limited to varicella, rubella, syphilis, parvovirus, cytomegalovirus (CMV), and toxoplasmosis. Certain maternal conditions such as diabetes and phenylketonuria (PKU) may also affect organs and other parts of the embryo during this developmental period. Drugs such as antiseizure medication and some antibiotics as well as chemicals including lead, mercury, tobacco, and alcohol also may result in structural and functional abnormalities.

Nurses should be aware that the biophysical profile (BPP): a. Is an accurate indicator of impending fetal well-being b. Is a compilation of health risk factors of the mother during the later stages of pregnancy c. Consists of a Doppler blood flow analysis and an amniotic fluid index (AFI) d. Involves an invasive form of ultrasonic examination

Is an accurate indicator of impending fetal well-being An abnormal BPP score is one indication that labor should be induced. The BPP evaluates the health of the fetus. The BPP requires many different measures. The BPP is a noninvasive procedure.

A first-time mother at 18 weeks of gestation is in for her regularly scheduled prenatal visit. The client tells the nurse that she is afraid that she is going into premature labor because she is beginning to have regular contractions. The nurse explains that this is the Braxton Hicks sign and teaches the client that this type of contraction: a. Is painless b. Increases with walking c. Causes cervical dilation d. Impedes oxygen flow to the fetus

Is painless Soon after the fourth month of gestation, uterine contractions can be felt through the abdominal wall. Braxton Hicks contractions are regular and painless and continue throughout the pregnancy. Although they are not painful, some women complain that they are annoying.

In order to reassure and educate pregnant clients about changes in their breasts, nurses should be aware that: a. The visibility of blood vessels that form an intertwining blue network indicates full function of Montgomerys tubercles and possibly infection of the tubercles b. The mammary glands do not develop until 2 weeks before labor c. Lactation is inhibited until the estrogen level declines after birth d. Colostrum is the yellowish oily substance used to lubricate the nipples for breastfeeding

Lactation is inhibited until the estrogen level declines after birth Lactation is inhibited until after birth. The visible blue network of blood vessels is a normal outgrowth of a richer blood supply. The mammary glands are functionally complete by midpregnancy. Colostrum is a creamy white-to-yellow premilk fluid that can be expressed from the nipples before birth.

With regard to the development of the respiratory system, maternity nurses should be aware that: a. The respiratory system does not begin developing until after the embryonic stage b. The infants lungs are considered mature when the L/S ratio is 1:1, at about 32 weeks c. Maternal hypertension can reduce maternal-placental blood flow, accelerating lung maturity d. Fetal respiratory movements are not visible on ultrasound scans until at least 16 weeks

Maternal hypertension can reduce maternal-placental blood flow, accelerating lung maturity A reduction in placental blood flow stresses the fetus, increases blood levels of corticosteroids, and thus accelerates lung maturity. Development of the respiratory system begins during the embryonic phase and continues into childhood. The infants lungs are mature when the L/S ratio is 2:1, at about 35 weeks. Lung movements have been seen on ultrasound scans at 11 weeks.

A 30-year-old gravida 3, para 2-0-0-2 is at 18 weeks of gestation. What screening test should be suggested to her? a. Biophysical profile b. Chorionic villi sampling c. Maternal serum alpha-fetoprotein (MSAFP) screening d. Screening for diabetes mellitus

Maternal serum alpha-fetoprotein (MSAFP) screening The biochemical assessment MSAFP test is performed from week 15 to week 20 of gestation (weeks 16 to 18 are ideal). A biophysical profile is a method of biophysical assessment of fetal well-being in the third trimester. Chorionic villi sampling is a biochemical assessment of the fetus that should be performed from the tenth to twelfth weeks of gestation. Screening for diabetes mellitus begins with the first prenatal visit.

Which statement about a condition of pregnancy is accurate? a. Insufficient salivation (ptyalism) is caused by increases in estrogen. b. Acid indigestion (pyrosis) begins early but declines throughout pregnancy. c. Hyperthyroidism often develops (temporarily) because hormone production increases. d. Nausea and vomiting rarely have harmful effects on the fetus and may be beneficial.

Nausea and vomiting rarely have harmful effects on the fetus and may be beneficial. Normal nausea and vomiting rarely produce harmful effects and may be less likely to result in miscarriage or preterm labor. Ptyalism is excessive salivation that may be caused by a decrease in unconscious swallowing or by stimulation of the salivary glands. Pyrosis begins as early as the first trimester and intensifies through the third trimester. Increased hormone production does not lead to hyperthyroidism in pregnant women.

A woman arrives at the clinic for a pregnancy test. Her last menstrual period (LMP) was February 14, 2011. Her expected date of birth (EDB) is: a. September 17, 2011 b. November 7, 2011 c. November 21, 2011 d. December 17, 2011

November 21, 2011 Using Ngeles rule, the EDB is calculated by subtracting 3 months from the month of the LMP and adding 7 days + 1 year to the day of the LMP. Therefore, with an LMP of February 14, 2011, her due date is November 21, 2011.

During the initial visit with a client who is beginning prenatal care, nurses should be aware that: a. The first interview is a relaxed, get-acquainted affair in which nurses gather some general impressions b. If nurses observe handicapping conditions, they should be sensitive and not inquire about them because the client will do that in her own time c. Nurses should be alert to the appearance of potential parenting problems, such as depression or lack of family support d. Because of legal complications, nurses should not ask about illegal drug use; that is left to physicians

Nurses should be alert to the appearance of potential parenting problems, such as depression or lack of family support Besides these potential problems, nurses need to be alert to the womans attitude toward keeping regular health care appointments. If the client lacks insurance, the nurse may be able to direct her to resources that provide assistance for pregnant women (i.e., Women, Infants, and Children [WIC], Medicaid).

Which assessment is not included in the fetal biophysical profile (BPP)? a. Fetal movement b. Fetal tone c. Fetal heart rate d. Amniotic fluid index e. Placental grade

Placental grade Fetal movement, tone, heart rate, and amniotic fluid index are all assessed in a BPP.

Risk factors tend to be interrelated and cumulative in their effect. While planning the care for a laboring client with diabetes mellitus, the nurse is aware that she is at a greater risk for: a. Oligohydramnios b. Polyhydramnios c. Postterm pregnancy d. Chromosomal abnormalities

Polyhydramnios Polyhydramnios or amniotic fluid in excess of 2000 ml is 10 times more likely to occur in diabetic rather than nondiabetic pregnancies. This puts the mother at risk for premature rupture of membranes, premature labor, and postpartum hemorrhage. Prolonged rupture of membranes, intrauterine growth restriction (IUGR), intrauterine fetal death, and renal agenesis (Potter syndrome) put the client at risk for developing oligohydramnios. Anencephaly, placental insufficiency, and perinatal hypoxia contribute to the risk for postterm pregnancy. Maternal age greater than 35 and balanced translocation (maternal and paternal) are risk factors for chromosomal abnormalities.

When evaluating a client whose primary complaint is amenorrhea, the nurse must be aware that lack of menstruation is most often the result of: a. Stress b. Excessive exercise c. Pregnancy d. Eating disorders

Pregnancy Amenorrhea, or the absence of menstrual flow, is most often a result of pregnancy. Although stress may be a contributing factor to amenorrhea, pregnancy is the most common cause. Although excessive exercise may be a contributing factor to amenorrhea, pregnancy is the most common cause. Although eating disorders may be a contributing factor to amenorrhea, pregnancy is the most common cause.

A 31-year-old woman believes that she may be pregnant. She took an over-the-counter (OTC) pregnancy test 1 week ago after missing her period; the test was positive. During her assessment interview, the nurse inquires about the womans last menstrual period (LMP) and asks whether she is taking any medications. The woman states that she takes medicine for epilepsy. She has been under considerable stress lately at work and has not been sleeping well. She also has a history of irregular periods. Her physical examination does not indicate that she is pregnant. She has an ultrasound scan, which reveals that she is not pregnant. What is the most likely cause of the false-positive pregnancy test result? a. She took the pregnancy test too early. b. She takes anticonvulsants. c. She has a fibroid tumor. d. She has been under considerable stress and has a hormone imbalance.

She takes anticonvulsants. Anticonvulsants may cause false-positive pregnancy test results. OTC pregnancy tests use enzyme-linked immunosorbent assay (ELISA) technology, which can yield positive results as soon as 4 days after implantation. Implantation occurs 6 to 10 days after conception. If the woman were pregnant, she would be into her third week at this point (having missed her period 1 week ago). Fibroid tumors do not produce hormones and have no bearing on human chorionic gonadotropin (hCG) pregnancy tests. Although stress may interrupt normal hormone cycles (menstrual cycles), it does not affect hCG levels or produce positive pregnancy test results.

A woman is at 14 weeks of gestation. The nurse expects to palpate the fundus at which level? a. Not palpable above the symphysis at this time b. Slightly above the symphysis pubis c. At the level of the umbilicus d. Slightly above the umbilicus

Slightly above the symphysis pubis In normal pregnancies the uterus grows at a predictable rate. It may be palpated above the symphysis pubis sometime between the twelfth and fourteenth weeks of pregnancy. As the uterus grows it may be palpated above the symphysis pubis sometime between the twelfth and fourteenth weeks of pregnancy. At 14 weeks the uterus is not yet at the level of the umbilicus. The fundus is not palpable above the umbilicus until 22 to 24 weeks of gestation.

When nurses help their expectant mothers assess the daily fetal movement counts (DFMCs) they should be aware that: a. Alcohol or cigarette smoke can irritate the fetus into greater activity b. Kick counts should be taken every half hour and averaged every 6 hours, with every other 6-hour stretch off c. The fetal alarm signal should go off when fetal movements stop entirely for 12 hours d. Obese mothers familiar with their bodies can assess fetal movement as well as average-sized women

The fetal alarm signal should go off when fetal movements stop entirely for 12 hours No movement in a 12-hour period is cause for investigation and possibly intervention. Alcohol and cigarette smoke temporarily reduce fetal movement. The mother should count fetal activity (kick counts) two or three times daily for 60 minutes each time. Obese women have a harder time assessing fetal movement.

With regard to the initial physical exam of a woman beginning prenatal care, maternity nurses should be aware that: a. Only women who show physical signs or meet the sociologic profile should be assessed for physical abuse b. The woman should empty her bladder before the pelvic examination c. The distribution, amount, and quality of body hair are of no particular importance d. The size of the uterus is discounted in the initial examination because it is just going to get bigger soon

The woman should empty her bladder before the pelvic examination An empty bladder facilitates the examination; also, this is an opportunity to get a urine sample easily for a number of tests. All women should be assessed for a history of physical abuse, particularly because the likelihood of abuse increases during pregnancy. Noting body hair is important because body hair reflects nutritional status, endocrine function, and hygiene. Particular attention is paid to the size of the uterus because it is an indication of the duration of gestation.

In order to reassure and educate pregnant clients about changes in the uterus, nurses should be aware that: a. Lightening occurs near the end of the second trimester as the uterus rises into a different position b. The womans increased urinary frequency in the first trimester is the result of exaggerated uterine antireflexion caused by softening c. Braxton Hicks contractions become more painful in the third trimester, particularly if the woman tries to exercise d. The uterine souffle is the movement of the fetus

The womans increased urinary frequency in the first trimester is the result of exaggerated uterine antireflexion caused by softening The softening of the lower uterine segment is called the Hegar sign. Lightening occurs in the last 2 weeks of pregnancy, when the fetus descends. Braxton Hicks contractions become more defined in the final trimester but are not painful. Walking or exercise usually causes them to stop. The uterine souffle is the sound made by blood in the uterine arteries; it can be heard with a fetal stethoscope.

A woman is in her seventh month of pregnancy. She has been complaining of nasal congestion and occasional epistaxis. The nurse suspects that: a. This is a normal respiratory change in pregnancy caused by elevated levels of estrogen b. This is an abnormal cardiovascular change and the nosebleeds are an ominous sign c. The woman is a victim of domestic violence and is being hit in the face by her partner d. The woman has been using cocaine intranasally

This is a normal respiratory change in pregnancy caused by elevated levels of estrogen Elevated levels of estrogen cause capillaries to become engorged in the respiratory tract, which may result in edema in the nose, larynx, trachea, and bronchi. This congestion may cause nasal stuffiness and epistaxis. Cardiovascular changes in pregnancy may cause edema in lower extremities. Domestic violence cannot be determined based on the sparse facts provided. If the woman had been hit in the face, she most likely would have additional physical findings. Cocaine use cannot be determined based on the sparse facts provided.

In her work with pregnant women of various cultures, a nurse practitioner has observed various practices that seemed strange or unusual. She has learned that cultural rituals and practices during pregnancy seem to have one purpose in common. Which statement best describes that purpose? a. To promote family unity b. To ward off the evil eye c. To appease the gods of fertility d. To protect the mother and fetus during pregnancy

To protect the mother and fetus during pregnancy Although many cultures consider pregnancy normal, certain practices are expected of women of all cultures to ensure a good outcome. Cultural prescriptions tell women what to do, and cultural proscriptions establish taboos. The purposes of these practices are to prevent maternal illness resulting from a pregnancy-induced imbalanced state and to protect the vulnerable fetus.

A nurse should be aware that a partners main role in pregnancy is: a. To provide financial support b. To protect the pregnant woman from old wives tales c. To support and nurture the pregnant woman d. To make sure the pregnant woman keeps prenatal appointments

To support and nurture the pregnant woman The partners main role in pregnancy is to nurture the pregnant woman and respond to her feelings of vulnerability. Although financial support is important, it is not the partners main role in pregnancy. Protecting the pregnant woman from old wives tales is not the partners role. The womans partner can encourage the client to keep all appointments; however, this is not the most important role during the pregnancy.

When instructing a pregnant client regarding personal hygiene, it is important for a maternity nurse to be aware that: a. Tub bathing is permitted even in late pregnancy unless membranes have ruptured b The perineum should be wiped from back to front c. Bubble bath and bath oils are permissible because they add an extra soothing and cleansing action to the bath d. Expectant mothers should use specially treated soap to cleanse the nipples

Tub bathing is permitted even in late pregnancy unless membranes have ruptured The main danger from taking baths is falling in the tub.

A maternal serum alpha-fetoprotein (MSAFP) test indicates an elevated level. It is repeated and again is reported as higher than normal. What is the next step in the assessment sequence to determine the well-being of the fetus? a. Percutaneous umbilical blood sampling (PUBS) b. Ultrasound for fetal anomalies c. Biophysical profile (BPP) for fetal well-being d. Amniocentesis for genetic anomalies

Ultrasound for fetal anomalies If AFP findings are abnormal, follow-up procedures include genetic counseling for families with a history of neural tube defect (NTD), repeated AFP, ultrasound examination, and possibly amniocentesis. Indications for use of PUBS include prenatal diagnosis of inherited blood disorders, karyotyping of malformed fetuses, detection of fetal infection, determination of the acid-base status of fetuses with intrauterine growth restriction (IUGR), and assessment and treatment of isoimmunization and thrombocytopenia in the fetus. A BPP is a method of assessing fetal well-being in the third trimester. Prior to an amniocentesis, the client would have an ultrasound for direct visualization of the fetus.

Signs and symptoms that a woman should report immediately to her health care provider include (choose all that apply): a. Vaginal bleeding b. Rupture of membranes c. Heartburn accompanied by severe headache d. Decreased libido e.Urinary frequency

Vaginal bleeding Rupture of membranes Heartburn accompanied by severe headache Vaginal bleeding, rupture of membranes, and severe headaches are signs of potential complications in pregnancy. Clients should be advised to report these signs to their health care provider.

A pregnant couple has formulated a birth plan and is reviewing it with the nurse at an expectant parents class. Which aspect of their birth plan would be considered unrealistic and require further discussion with the nurse? a. My husband and I have agreed that my sister will be my coach because he becomes anxious with regard to medical procedures and blood. He will be nearby and check on me every so often to make sure everything is okay. b. We plan to use the techniques taught in the Lamaze classes to reduce the pain experienced during labor. c. We want the labor and birth to take place in a birthing room. My husband will come in the minute the baby is born. d. We do not want the fetal monitor used during labor because it will interfere with movement and doing effleurage.

We do not want the fetal monitor used during labor because it will interfere with movement and doing effleurage. Because monitoring is essential to assess fetal well-being, it is not a factor that can be determined by the couple. The nurse should fully explain its importance. The option for intermittent electronic monitoring could be explored if this is a low risk pregnancy and as long as labor is progressing normally. The birth plan is a tool with which parents can explore their childbirth options; however, the plan must be viewed as tentative.

Many pregnant women have questions regarding work and travel during pregnancy. Nurse should instruct clients that: a. Women should sit for as long as possible and cross their legs at the knees from time to time for exercise b. Women should avoid seat belts and shoulder restraints in the car because they press on the fetus c. Metal detectors at airport security checkpoints can harm the fetus if the woman passes through them a number of times d. While working or traveling in a car or on a plane, women should arrange to walk around at least every hour or so

While working or traveling in a car or on a plane, women should arrange to walk around at least every hour or so Periodic walking helps prevent thrombophlebitis.

A nurse should advise which women about continued condom use during pregnancy? a. Unmarried pregnant women b. Women at risk for acquiring or transmitting sexually transmitted infections (STIs) c. All pregnant women d. Women at risk for candidiasis

Women at risk for acquiring or transmitting sexually transmitted infections (STIs) The objective of safer sex is to provide prophylaxis against the acquisition and transmission of STIs. Because these diseases may be transmitted to the woman and her fetus, condom use is recommended throughout pregnancy if the woman is at risk for acquiring an STI.

The measurement of lecithin in relation to sphingomyelin (L/S ratio) is used to determine fetal lung maturity. Which ratio reflects maturity of the lungs? a. 1.4:1 b. 1.8:1 c. 2:1 d. 1:1

2:1 This ratio indicates a 2:1 ratio of lecithin to sphingomyelin, an indicator of lung maturity. 1.4:1 ratio indicates immaturity of the fetal lungs. 1.8:1 ratio indicates immaturity of the fetal lungs. 1:1 ratio indicates immaturity of the fetal lungs.

The nurse should know that once the human immunodeficiency virus (HIV) enters the body, seroconversion to HIV positivity usually occurs within: a. 6 to 10 days b. 2 to 4 weeks c. 6 to 12 weeks d. 6 months

6 to 12 weeks Seroconversion to HIV positivity usually occurs within 6 to 12 weeks after the virus has entered the body. Six to 10 days is too short a time period for seroconversion to HIV positivity to occur. Two to 4 weeks is too short a time period for seroconversion to HIV positivity to occur. Seroconversion to HIV positivity usually occurs within 6 to 12 weeks after the virus has entered the body; 6 months is too long.

Which woman is at high risk for psychologic complications after hysterectomy? a. A 55-year-old woman who has been having abnormal bleeding and pain for 3 years b. A 46-year-old woman who has three children and has just been promoted at work c. A 62-year-old widow who has three friends who have had uncomplicated hysterectomies d. A 19-year-old woman who had a ruptured uterus after giving birth to her first child

A 19-year-old woman who had a ruptured uterus after giving birth to her first child The 19-year-old woman is still in her childbearing years. Often the uterus is related to self-concept in women of this age-group, and they may feel that sexual functioning is related to having a uterus. The 55-year-old woman is past her childbearing years and has had bleeding and pain for 3 years. The surgery may be well received as a method of pain relief. The 46-year-old woman has a family and has positive events occurring in her life (job promotion). The 62-year-old woman is past her reproductive years and has relationships with others who have had positive outcomes.

In the acronym BRAIDED, which letter is used to identify the key components of informed consent that the nurse must document? a. B stands for babies. b. R stands for reproduction. c. A stands for alternatives. d. I stands for ineffective.

A stands for alternatives. A stands for alternatives, or information about other viable methods. B stands for benefits, or information about advantages and success rates. R stands for risks, or information about disadvantages and failure rates. I stands for inquiries, or the opportunity to ask questions.

A woman is 16 weeks pregnant and has elected to terminate her pregnancy. The nurse knows that the most common technique used for medical termination of a pregnancy in the second trimester is: a. Administration of prostaglandins b. Instillation of hypertonic saline into the uterine cavity c. IV administration of Pitocin d. Vacuum aspiration

Administration of prostaglandins The most common technique for medical termination of a pregnancy in the second trimester is administration of prostaglandins. Hypertonic solutions injected directly into the uterus account for less than 1% of all abortions because other methods are safer and easier to use. IV administration of Pitocin is used to induce labor in a woman with a third-trimester fetal demise. Vacuum aspiration is used for abortions in the first trimester.

A woman had unprotected intercourse 36 hours ago and is concerned that she may become pregnant because it is her fertile time. She asks the nurse about emergency contraception. The nurse would tell her that: a. It is too late; she needed to begin treatment within 24 hours after intercourse b. Preven, an emergency contraceptive method, is 98% effective at preventing pregnancy c. An over-the-counter antiemetic can be taken 1 hour before each contraceptive dose to prevent nausea and vomiting d. The most effective approach is to use a progestin-only preparation

An over-the-counter antiemetic can be taken 1 hour before each contraceptive dose to prevent nausea and vomiting To minimize the side effect of nausea that occurs with high doses of estrogen and progestin, the woman can take an over-the-counter antiemetic 1 hour before each dose. Emergency contraception is used within 72 hours of unprotected intercourse to prevent pregnancy. Postcoital contraceptive use is 74% to 90% effective at preventing pregnancy. Oral emergency contraceptive regimens may include progestin-only and estrogen-progestin pills. Women with contraindications to estrogen use should use progestin-only pills.

A woman presents with a possible diagnosis of polycystic ovary syndrome (PCOS). While completing the initial assessment of the client, the nurse understands that clinical manifestations of PCOS might include allexcept: a. Anorexia b. Hirsutism c. Irregular menses d. Infertility

Anorexia These clients often present with obesity rather than anorexia and weight loss. 40% of these women also display glucose intolerance and hyperinsulinemia. Excessive hair growth is often present with PCOS. This client is likely to have irregular menses or even amenorrhea. Infertility as a result of decreased levels of follicle-stimulating hormone is common with this syndrome.

With regard to the structure and function of the placenta, the maternity nurse should be aware that: a. As the placenta widens, it gradually thins to allow easier passage of air and nutrients b. As one of its early functions, the placenta acts as an endocrine gland c. The placenta is able to keep out most potentially toxic substances, such as cigarette smoke, to which the mother is exposed d. Optimal blood circulation is achieved through the placenta when the woman is lying on her back or standing

As one of its early functions, the placenta acts as an endocrine gland The placenta produces four hormones necessary to maintain the pregnancy. The placenta widens until week 20 and continues to grow thicker. Toxic substances such as nicotine and carbon monoxide readily cross the placenta into the fetus. Optimal circulation occurs when the woman is lying on her side.

A pregnant woman at 25 weeks of gestation tells the nurse that she dropped a pan last week and her baby jumped at the noise. Which response by the nurse is most accurate? a. That must have been a coincidence; babies cant respond like that. b. The fetus is demonstrating the aural reflex. c. Babies respond to sound starting at about 24 weeks of gestation. d. Let me know if it happens again; we need to report that to your midwife.

Babies respond to sound starting at about 24 weeks of gestation. Babies respond to sound starting at about 24 weeks of gestation. Fetuses can respond to sound by 24 weeks or so. Acoustic stimulations can evoke a fetal heart rate response. There is no such thing as an aural reflex. Stating that a report needs to be made if the fetus moves again at a noise gives the impression that something is wrong.

On vaginal examination of a 30-year-old woman, the nurse documents the following findings: profuse, thin, grayish-white vaginal discharge with a fishy odor; complains of pruritus. Based on these findings, the nurse suspects that this woman has: a. Bacterial vaginosis b. Candidiasis c. Trichomoniasis d. Gonorrhea

Bacterial vaginosis Most women with bacterial vaginosis (BV) complain of a characteristic fishy odor. The discharge usually is profuse, thin, and has a white, gray, or milky color. Some women also may have mild irritation or pruritus. The discharge associated with candidiasis is thick, white, and lumpy and resembles cottage cheese. Trichomoniasis may be asymptomatic, but women commonly have a characteristic yellowish to greenish, frothy, mucopurulent, copious, and malodorous discharge. Women with gonorrhea are often asymptomatic. They may have a purulent endocervical discharge, but discharge usually is minimal or absent.

Care management of a woman diagnosed with acute pelvic inflammatory disease (PID) most likely includes: a. Oral antiviral therapy b. Bed rest in a semi-Fowler position c. Antibiotic regimen continued until symptoms subside d. Frequent pelvic examination to monitor the progress of healing

Bed rest in a semi-Fowler position The woman with acute PID should be on bed rest in a semi-Fowler position. Broad-spectrum antibiotics are used. Antibiotics must be taken as prescribed, even if symptoms subside. Few pelvic examinations should be conducted during the acute phase of the disease.

The prevalence of urinary incontinence (UI) increases as women age, with more than one third of women in the United States suffering from some form of this disorder. The symptoms of mild to moderate UI can be successfully decreased by a number of strategies. Which of these should the nurse instruct the client to use first? a. Pelvic floor support devices b. Bladder training and pelvic muscle exercises c. Surgery d. Medications

Bladder training and pelvic muscle exercises Pelvic muscle exercises, known as Kegel exercises, along with bladder training can significantly decrease or entirely relieve stress incontinence in many women. Pelvic floor support devices also known as pessaries come in a variety of shapes and sizes. Pessaries may not be effective for all women and require scrupulous cleaning to prevent infection. Anterior and posterior repairs and even a hysterectomy may be performed. If surgical repair is performed, the nurse must focus her care on preventing infection and helping the woman avoid putting stress on the surgical site. Pharmacologic therapy includes selective serotonin-norepinephrine reuptake inhibitors or vaginal estrogen therapy. These are not the first action a nurse should recommend.

When discussing estrogen replacement therapy (ERT) with a perimenopausal woman, the nurse includes the risk of: a. Breast cancer b. Vaginal and urinary tract atrophy c. Osteoporosis d. Arteriosclerosis

Breast cancer Women with a high risk for breast cancer should be counseled against using ERT. Estrogen prevents atrophy of vaginal and urinary tract tissue. Estrogen protects against the development of osteoporosis. Estrogen has a favorable effect on circulating lipids, reducing low-density lipoprotein (LDL) and total cholesterol and increasing high-density lipoprotein (HDL). It also has a direct antiatherosclerotic effect on the arteries.

Which medications can be taken by postmenopausal women to treat and/or prevent osteoporosis? Choose all that apply. a. Calcium b. Evista c. Fosamax d. Actonel e. Calcitonin

Calcium Evista Fosamax Actonel Calcitonin Calcium, Evista, Fosamax, Actonel, and Calcitonin can be used by postmenopausal women to treat or prevent osteoporosis.

Nafarelin (Synarel) is used to treat mild to severe endometriosis. The nurse should tell the woman taking this medication that the drug: a. Stimulates the secretion of gonadotropin-releasing hormone (GnRH), thereby stimulating ovarian activity b. Should be sprayed into one nostril every other day c. Should be injected into subcutaneous tissue bid d. Can cause her to experience some hot flashes and vaginal dryness

Can cause her to experience some hot flashes and vaginal dryness Nafarelin is a GnRH agonist, and its side effects are similar to those of menopause. The hypoestrogenism effect results in hot flashes and vaginal dryness. Nafarelin is a GnRH agonist that suppresses the secretion of GnRH. Nafarelin is administered twice daily by nasal spray. Nafarelin is administered intranasally.

What is the most common reproductive tract cancer associated with pregnancy? a. Cervical cancer b. Uterine cancer c. Ovarian cancer d. Fallopian tube cancer

Cervical cancer The incidence of cervical cancer concurrent with pregnancy is reported to be 3%, making it the most common reproductive tract cancer associated with pregnancy. Uterine cancer is rarely diagnosed during pregnancy. Ovarian cancer is the second most frequent cancer diagnosis in pregnancy. At approximately 1% it remains a rare occurrence. The peak incidence of tubal cancer is between the ages of 50 and 55. For this cancer to be concurrent with pregnancy is only a remote possibility.

Chapter 6: Reproductive System Concerns Chapter 7: Sexually Transmitted & Other Infections Chapter 8: Contraception & Abortion Chapter 9: Infertility Chapter 11: Structural Disorders and Neoplasms of the Reproductive System

Chapter 12: Conception and Fetal Development Chapter 13: Anatomy and Physiology of Pregnancy Chapter 15: Nursing Care of the Family During Pregnancy Chapter 26: Assessment for Risk Factors in Pregnancy

Many factors, male and female, contribute to normal fertility. Approximately 40% of cases of infertility are related to the female partner. These alterations in physiology belong to one of five categories. Choose all that apply. a. Congenital or developmental factors b. Hormonal or ovulatory factors c. Tubal or peritoneal factors d. Uterine factors e. Emotional or psychologic factors

Congenital or developmental factors Hormonal or ovulatory factors Tubal or peritoneal factors Uterine factors Female infertility can be attributed to alterations in any one of these systems along with possible vaginal-cervical factors. The diagnosis and treatment of infertility require considerable emotional investment and may cause psychologic stress. However, this is not considered one of the factors associated with infertility. Feelings connected with infertility are many and complex. Resolve is an organization that provides support, advocacy, and education for both clients and health care providers.

Which suggestion is appropriate for a client who complains of hot flashes? Choose all that apply. a. Consume large quantities of caffeine b. Drink a glass of wine c. Eat Mexican food d. Drink ice water e. Drink warm beverages

Drink ice water Ice water may help alleviate the hot flashes. Consuming large quantities of caffeine, drinking a glass of wine, eating Mexican food, or drinking warm beverages most likely would exacerbate the hot flashes.

During her annual gynecologic checkup, a 17-year-old woman states that recently she has been experiencing cramping and pain during her menstrual periods. The nurse documents this complaint as: a. Amenorrhea b. Dysmenorrhea c. Dyspareunia d. Premenstrual syndrome (PMS)

Dysmenorrhea Dysmenorrhea is pain during or shortly before menstruation. Amenorrhea is the absence of menstrual flow. Dyspareunia is pain during intercourse. PMS is a cluster of physical, psychologic, and behavioral symptoms that begin in the luteal phase of the menstrual cycle and resolve within a couple of days of the onset of menses.

Nurses who provide health care for women should recognize that the most commonly reported gynecologic problem for women of any age-group is: a. Dysmenorrhea b. Menorrhagia c. Dyspareunia d. Endometriosis

Dysmenorrhea Dysmenorrhea, or pain during or shortly before menstruation, is one of the most common gynecologic problems in women of all ages. Menorrhagia, or excessive bleeding, is an alteration in cyclic bleeding; it is not the most commonly reported gynecologic problem. Dyspareunia, or painful intercourse, is commonly associated with endometriosis. Endometriosis is a type of menstrual disorder, but it is not the most commonly reported gynecologic problem.

The nurse knows that teaching about external radiation therapy is effective when the woman: a. Uses ointment to keep her skin from drying out b. Washes the irradiated area with deodorant soap c. Eats a diet high in protein and drinks at least 2000 ml of fluid a day d. Washes off the markings for the radiation site after each treatment

Eats a diet high in protein and drinks at least 2000 ml of fluid a day To maintain good nutrition, the woman should eat high-protein meals or use protein supplements and should have a high daily fluid intake of 2 to 3 L. The woman is counseled about good skin care and taught to avoid soaps, ointments, cosmetics, and deodorants because these may contain metals that would alter the radiation dose she receives. The woman is counseled about good skin care and taught to avoid soaps, ointments, cosmetics, and deodorants because these may contain metals that would alter the radiation dose she receives. Markings may be made to indicate the exact location needed for irradiation. These should remain until treatment is complete.

Which test used to diagnose the basis of infertility is done during the luteal or secretory phase of the menstrual cycle? a. Hysterosalpingogram b. Endometrial biopsy c. Laparoscopy d. Follicle-stimulating hormone (FSH) level

Endometrial biopsy Endometrial biopsy is scheduled after ovulation, during the luteal phase of the menstrual cycle. A hysterosalpingogram is scheduled 2 to 5 days after menstruation to avoid flushing a potentially fertilized ovum out through a uterine tube into the peritoneal cavity. Laparoscopy usually is scheduled early in the menstrual cycle. Hormone analysis is performed to assess endocrine function of the hypothalamic-pituitary-ovarian axis when menstrual cycles are absent or irregular.

Danazol (androgens) may be prescribed for an infertile woman if she has: a. Thyroid dysfunction b. Elevated levels of prolactin c. Inadequate levels of follicle-stimulating hormone (FSH) d. Endometriosis

Endometriosis Danazol treats endometriosis by suppressing ovarian activity and eliminating stimulation of endometrial tissue. Thyroid dysfunction is not treated with danazol. Danazol is not an appropriate treatment for a client with elevated levels of prolactin. Inadequate levels of FSH do not respond to treatment with danazol.

The conscious decision on when to conceive or avoid pregnancy throughout the reproductive years is called: a. Family planning b. Birth control c. Contraception d. Assisted reproductive therapy

Family planning Family planning is the process of deciding when and if to have children. Birth control is the device and/or practice used to reduce the risk of conceiving or bearing children. Contraception is the intentional prevention of pregnancy during sexual intercourse. Assisted reproductive therapy is one of several possible treatments for infertility.

Examples of sexual risk behaviors associated with exposure to a sexually transmitted infection (STI) include (choose all that apply): a. Fellatio b. Unprotected anal intercourse c. Multiple sex partners d. Dry kissing e. Abstinence

Fellatio Unprotected anal intercourse Multiple sex partners Engaging in fellatio, unprotected anal intercourse, or having multiple sex partners increases the exposure risk and the possibility of acquiring an STI. Dry kissing and abstinence are considered safe sexual practices.

A couple comes in for an infertility workup, having attempted to get pregnant for 2 years. The woman, 37, has always had irregular menstrual cycles but is otherwise healthy. The man has fathered two children from a previous marriage and had a vasectomy reversal 2 years ago. The man has had two normal semen analyses, but the sperm seem to be clumped together. What additional test is needed? a. Testicular biopsy b. Antisperm antibodies c. Follicle-stimulating hormone (FSH) level d. Examination for testicular infection

Follicle-stimulating hormone (FSH) level The woman has irregular menstrual cycles. The scenario does not indicate that she has had any testing related to this irregularity. Hormone analysis is performed to assess endocrine function of the hypothalamic-pituitary-ovarian axis when menstrual cycles are absent or irregular. Determination of blood levels of prolactin, FSH, luteinizing hormone (LH), estradiol, progesterone, and thyroid hormones may be necessary to diagnose the cause of irregular menstrual cycles. A testicular biopsy is indicated only in cases of azoospermia (no sperm cells) or severe oligospermia (low number of sperm cells). Antisperm antibodies are produced by a man against his own sperm. This is unlikely to be the case here because the husband has already produced children. Examination for testicular infection would be done before semen analysis. Furthermore, infection would affect spermatogenesis.

Which sexually transmitted infection is not bacterial and thus not treatable with antibiotics? a. Chlamydia b. Gonorrhea c. Genital herpes d. Syphilis

Genital herpes Genital herpes is a viral infection and therefore does not respond to antibiotics. Chlamydia is a bacterial infection and is treated with doxycycline or azithromycin. Gonorrhea is a bacterial infection and is treated with any of several antibiotics. Syphilis is a bacterial infection and is treated with penicillin.

A couple is trying to cope with an infertility problem. They want to know what they can do to preserve their emotional equilibrium. The nurses most appropriate response is: a. Tell your friends and family so that they can help you. b. Talk only to other friends who are infertile, because only they can help. c. Get involved with a support group. Ill give you some names. d. Start adoption proceedings immediately, because obtaining an infant is very difficult.

Get involved with a support group. Ill give you some names. Venting negative feelings may unburden the couple. A support group may provide a safe haven for the couple to share their experiences and gain insight from others experiences. Although talking about their feelings may unburden them of negative feelings, infertility can be a major stressor that affects the couples relationships with family and friends. Limiting their interactions to other infertile couples may be a beginning point for addressing psychosocial needs, but depending on where the other couple is in their own recovery process, this may or may not be of assistance to them. Telling the couple to start adoption proceedings immediately is not supportive of the psychosocial needs of this couple and may be detrimental to their well-being.

To detect the human immunodeficiency virus (HIV), most laboratory tests focus on: a. HIV virus b. HIV antibodies c. CD4 counts d. CD8 counts

HIV antibodies The screening tool used to detect HIV is the enzyme immunoassay, which tests for the presence of antibodies to the HIV. In order to determine if the HIV is present, the test performed must be able to detect antibodies to the virus, not the virus itself. CD4 counts are associated with the incidence of acquired immunodeficiency syndrome (AIDS) in HIV-infected individuals. CD8 counts are not performed in order to detect HIV.

Which statements are true? Choose all that apply. a. Human papillomavirus (HPV) infections are thought to be less common in pregnant women than in women who are not pregnant. b. HPV infections are thought to be more common in pregnant women than in women who are not pregnant. c. HPV infection previously was called genital warts. d. HPV infection previously was called herpes. e. HPV may cause cancer.

HPV infections are thought to be more common in pregnant women than in women who are not pregnant. HPV infection previously was called genital warts. HPV may cause cancer. HPV infections are thought to be more common in pregnant women, with an increase in incidence from the first trimester to the third trimester. HPV, formerly called venereal or genital warts, is a sexually transmitted infection with more than 30 known serotypes, several of which are associated with cervical cancer. HPV infections are thought to be more common in pregnant women than in women who are not pregnant. HPV formerly was called genital warts.

A 25-year-old single female comes to the gynecologists office for a follow-up visit related to her abnormal Pap smear. The test revealed that the client has human papillomavirus (HPV). The client asks, What is that? Can you get rid of it? Your best response is: a. Its just a little lump on your cervix. We can just freeze it off. b. HPV stands for human papillomavirus. It is a sexually transmitted infection that may lead to cervical cancer. c. HPV is a type of early human immunodeficiency virus. You will die from this. d. You probably caught this from your current boyfriend. He should get tested for this.

HPV stands for human papillomavirus. It is a sexually transmitted infection that may lead to cervical cancer. It is important to inform the client about STIs and the risks involved with HPV. The health care team has a duty to provide proper information to the client, including information related to sexually transmitted infections (STIs). HPV and HIV are viruses that can be transmitted sexually, but they are not the same virus. The onset of HPV can be insidious. Often STIs go unnoticed. Abnormal bleeding frequently is the initial symptom. The client may have had HPV before her current boyfriend. You cannot make any deductions from this limited information.

Injectable progestins (DMPA, Depo-Provera) are a good contraceptive choice for women who: a. Want menstrual regularity and predictability b. Have a history of thrombotic problems or breast cancer c. Have difficulty remembering to take oral contraceptives daily d. Are homeless or mobile and rarely receive health care

Have difficulty remembering to take oral contraceptives daily Advantages of DMPA include a contraceptive effectiveness comparable to that of combined oral contraceptives with the requirement of only four injections a year. Disadvantages of injectable progestins are prolonged amenorrhea and uterine bleeding. Use of injectable progestin carries an increased risk of venous thrombosis and thromboembolism. To be effective, DMPA injections must be administered every 11 to 13 weeks. Access to health care is necessary to prevent pregnancy or potential complications.

Which statement about the various forms of hepatitis is accurate? a. A vaccine exists for hepatitis C but not for hepatitis B. b. Hepatitis A is acquired by eating contaminated food or drinking polluted water. c. Hepatitis B is less contagious than human immunodeficiency virus (HIV). d. The incidence of hepatitis C is decreasing.

Hepatitis A is acquired by eating contaminated food or drinking polluted water. Contaminated milk and shellfish are common sources of infection with hepatitis A. A vaccine exists for hepatitis B but not for hepatitis C. Hepatitis B is more contagious than HIV. The incidence of hepatitis C is increasing.

Which virus is most threatening to the fetus and neonate? a. Hepatitis A virus b. Herpes simplex virus (HSV) c. Hepatitis B virus (HBV) d. Cytomegalovirus (CMV)

Hepatitis B virus (HBV) HBV is the virus most threatening to the fetus and neonate. Hepatitis A is not the most threatening to the fetus. HSV is not the most threatening to the neonate. Although serious, CMV is not the most threatening to the fetus.

Which viral sexually transmitted infection is characterized by a primary infection followed by recurrent episodes? a. Herpes simplex virus 2 (HSV-2) b. Human papillomavirus (HPV) c. Human immunodeficiency virus (HIV) d. Cytomegalovirus (CMV)

Herpes simplex virus 2 (HSV-2) The initial HSV genital infection is characterized by multiple painful lesions, fever, chills, malaise, and severe dysuria; it may last 2 to 3 weeks. Recurrent episodes of HSV infection commonly have only local symptoms that usually are less severe than those of the initial infection. With HPV infection, lesions are a chronic problem. HIV is a retrovirus. Seroconversion to HIV positivity usually occurs within 6 to 12 weeks after the virus has entered the body. Severe depression of the cellular immune system associated with HIV infection characterizes acquired immunodeficiency syndrome (AIDS). AIDS has no cure. In most adults, the onset of CMV infection is uncertain and asymptomatic. However, the disease may become a chronic, persistent infection.

When assessing a woman for menopausal discomforts, the nurse would expect the woman to describe the most frequently reported discomfort, which would be: a. Headaches b. Hot flashes c. Mood swings d. Vaginal dryness with dyspareunia

Hot flashes Vasomotor instability, in the form of hot flashes or flushing, is a result of fluctuating estrogen levels and is the most common disturbance of the perimenopausal woman. Headaches may be associated with a decline in hormone levels; however, it is not the most frequently reported discomfort for menopausal women. Mood swings may be associated with a decline in hormone levels; however, it is not the most frequently reported discomfort for menopausal women. Vaginal dryness and dyspareunia may be associated with a decline in hormone levels; however, it is not the most frequently reported discomfort for menopausal women.

A 48-year-old woman has just had a hysterectomy for endometrial cancer. Which statement alerts the nurse that further teaching is needed? a. I cant wait to go on the cruise that I have planned for this summer. b. I know that the surgery saved my life, but I will miss having sexual intercourse with my husband. c. I have asked my daughter to come and stay with me next week after I am discharged from the hospital. d. Well, I dont have to worry about getting pregnant anymore.

I know that the surgery saved my life, but I will miss having sexual intercourse with my husband. Stating that she will miss having sexual intercourse with her husband indicates that further teaching is needed for this client regarding sexual activities after a hysterectomy. Intercourse may be uncomfortable initially. The use of water-soluble lubricants, relaxation exercises, and changes in position may be helpful. Expressing plans for a vacation is a positive psychologic state with plans for the future. Stating that her daughter will stay with her indicates the client understands that she may need assistance during her acute recovery period. Stating that she no longer needs to worry about getting pregnant indicates knowledge related to the reproductive cycle and a positive outlook.

With regard to the use of intrauterine devices (IUDs), nurses should be aware that: a. Return to fertility can take several weeks after the device is removed b. IUDs containing copper can provide an emergency contraception option if inserted within a few days of unprotected intercourse c. IUDs offer the same protection against sexually transmitted infections as the diaphragm d. Consent forms are not needed for IUD insertion

IUDs containing copper can provide an emergency contraception option if inserted within a few days of unprotected intercourse The woman has up to 5 days to insert the IUD after unprotected sex. Return to fertility is immediate after removal of the IUD. IUDs offer no protection for sexually transmitted infections. A consent form is required for insertion, as is a negative pregnancy test.

In vitro fertilization-embryo transfer (IVF-ET) is a common approach for women with blocked fallopian tubes or with unexplained infertility and for men with very low sperm counts. A husband and wife have arrived for their preprocedural interview. The husband asks the nurse to explain what the procedure entails. The nurses most appropriate response is: a. IVF is a type of assisted reproductive therapy that involves collecting eggs from your wifes ovaries, fertilizing them in the lab with your sperm, and transferring the embryo to her uterus. b. A donor embryo will be transferred into your wifes uterus. c. Donor sperm will be used to inseminate your wife. d. Dont worry about the technical stuff; thats what we are here for.

IVF is a type of assisted reproductive therapy that involves collecting eggs from your wifes ovaries, fertilizing them in the lab with your sperm, and transferring the embryo to her uterus. A womans eggs are collected from her ovaries, fertilized in the laboratory with sperm, and transferred to her uterus after normal embryonic development has occurred. Transferring a donor embryo to the womans uterus describes the procedure for a donor embryo. Inseminating the woman with donor sperm describes therapeutic donor insemination. Telling the client not to worry discredits the clients need for teaching and is not the most appropriate response.

The most basic information a maternity nurse should have concerning conception is: a. Ova are considered fertile 48 to 72 hours after ovulation b. Sperm remain viable in the womans reproductive system for an average of 12 to 24 hours c. Conception is achieved when a sperm successfully penetrates the membrane surrounding the ovum d. Implantation in the endometrium occurs 6 to 10 days after conception

Implantation in the endometrium occurs 6 to 10 days after conception After implantation, the endometrium is called the decidua. Ova are considered fertile for about 24 hours after ovulation. Sperm remain viable in the womans reproductive system for an average of 2 to 3 days. Penetration of the ovum by the sperm is called fertilization. Conception occurs when the zygote, the first cell of the new individual, is formed.

You (the nurse) are reviewing the educational packet provided to a client about tubal ligation. What is an important fact you should point out? Choose all that apply. a. It is highly unlikely that you will become pregnant after the procedure. b. This is an effective form of 100% permanent sterilization. You wont be able to get pregnant. c. Sterilization offers some form of protection against sexually transmitted infections. d. Sterilization offers no protection against sexually transmitted infections. e. Your menstrual cycle will greatly increase after your sterilization.

It is highly unlikely that you will become pregnant after the procedure. Sterilization offers no protection against sexually transmitted infections. A woman is unlikely to become pregnant after tubal ligation. Sterilization offers no protection against sexually transmitted infections (STIs). Tubal ligation is not 100% effective. Tubal ligation does not offer any protection against STIs. Typically, the menstrual cycle remains the same after a tubal ligation.

A 23-year-old primiparous client with inconsistent prenatal care is admitted to the hospitals maternity unit in labor. The client states that she has tested positive for human immunodeficiency virus (HIV) but has not undergone any treatment during her pregnancy. As her primary nurse you understand that the risk of perinatal transmission can be significantly decreased by a number of prophylactic interventions. Select the appropriate interventions that should be included in this clients plan of care: a. Intrapartum treatment with antiviral medications b. Cesarean birth c. Postpartum treatment with antiviral medications d. Avoidance of breastfeeding e. Pneumococcal, hepatitis B, and Haemophilus influenzae vaccine

Intrapartum treatment with antiviral medications Cesarean birth Avoidance of breastfeeding The prophylactic measures of prenatal antiviral use, elective cesarean birth, and formula feeding reduce transmission as low as 1% to 2%. The client who refuses a cesarean birth should be given antiviral therapy intravenously during labor. Ideally, medications should be given prenatally. Administration of antiviral drugs in the postpartum period will not reduce transmission to the infant. All women who are HIV positive should be encouraged to receive these immunizations. They will not reduce the risk of perinatal transmission.

While instructing a couple regarding birth control, the nurse should be aware that the method called natural family planning: a. Is the same as coitus interruptus, or pulling out b. Uses the calendar method to align the womans cycle with the natural phases of the moon c. Is the only contraceptive practice acceptable to the Roman Catholic Church d. Relies on barrier methods during fertility phases

Is the only contraceptive practice acceptable to the Roman Catholic Church Natural family planning is the only contraceptive practice acceptable to the Roman Catholic Church. Pulling out is not the same as periodic abstinence, another name for natural family planning. The phases of the moon are not part of the calendar method or any method. Natural family planning is another name for periodic abstinence, which is the accepted way to pass safely through the fertility phases without relying on chemical or physical barriers.

With regard to endometriosis, nurses should be aware that: a. It is characterized by the presence and growth of endometrial tissue inside the uterus b. It is found more often in African-American women than in Caucasian or Asian women c.It may worsen with repeated cycles or remain as asymptomatic and disappear after menopause d. It is unlikely to affect sexual intercourse or fertility

It may worsen with repeated cycles or remain asymptomatic and disappear after menopause Symptoms vary among women, ranging from nonexistent to incapacitating. With endometriosis, the endometrial tissue is outside the uterus. Symptoms vary among women, ranging from nonexistent to incapacitating. Endometriosis is found equally in Caucasian and African-American women and is slightly more prevalent in Asian women. Women can experience painful intercourse and impaired fertility.

When assessing the client for amenorrhea, the nurse should be aware that this may be caused by all conditions except: a. Anatomic abnormalities b. Type 1 diabetes mellitus c. Lack of exercise d. Hysterectomy

Lack of exercise Lack of exercise is not a cause of amenorrhea. Strenuous exercise may cause amenorrhea. Anatomic abnormalities are a possible cause of amenorrhea. Type 1 diabetes mellitus is a possible cause of amenorrhea. Hysterectomy is a possible cause of amenorrhea.

During internal radiation therapy for cervical cancer, the nurse should: a. Wear gloves when assessing the cervical intracavity implant b. Instruct the client to urinate in the lead-lined bedpan or hat every 2 hours c. Prepare the client for an enema before insertion d. Limit staff or visitor exposure to 30 minutes or less per 8 hours

Limit staff or visitor exposure to 30 minutes or less per 8 hours Staff and visitor exposure should be limited to 30 minutes or less in an 8-hour period to reduce the risk of overexposure to radiation. Nurses need to protect themselves from overexposure to radiation. Wearing a shield is one method of protection. An indwelling catheter is inserted to prevent urinary distention that could dislodge the applicator. No bowel prep is necessary.

A 36-year-old woman has been diagnosed as having uterine fibroids. When planning care for this client, the nurse should know that: a. Fibroids are malignant tumors of the uterus that require radiation or chemotherapy b. Fibroids increase in size during the perimenopausal period c. Menorrhagia is a common finding d. The woman is unlikely to become pregnant as long as the fibroids are in her uterus

Menorrhagia is a common finding The major symptoms associated with fibroids are menorrhagia and the physical effects produced by large myomas. Fibroids are benign tumors of the smooth muscle of the uterus, and their etiology is unknown. Fibroids are estrogen sensitive and shrink as levels of estrogen decline. Fibroids occur in 25% of women of reproductive age and are seen in 2% of pregnant women.

One of the alterations in cyclic bleeding that occurs between periods is called: a. Oligomenorrhea b. Menorrhagia c. Leiomyoma d. Metrorrhagia

Metrorrhagia Metrorrhagia is bleeding between periods. It can be caused by progestin injections and implants. Oligomenorrhea is infrequent or scanty menstruation. Menorrhagia is excessive menstruation. Leiomyoma is a common cause of excessive bleeding.

Management of primary dysmenorrhea often requires a multifaceted approach. The nurse who provides care for a client with this condition should be aware that the optimal pharmacologic therapy for pain relief is: a. Acetaminophen b. Oral contraceptive pills (OCPs) c. Nonsteroidal antiinflammatory drugs (NSAIDs) d. Aspirin

Nonsteroidal antiinflammatory drugs (NSAIDs) NSAIDs have the strongest research results for pain relief. Often if one NSAID is not effective, another one will provide relief. Approximately 80% of women find relief from these prostaglandin inhibitors. Preparations containing acetaminophen are less effective for dysmenorrhea because they lack the antiprostaglandin properties of NSAIDs. OCPs are a reasonable choice for women who also want birth control. The benefit of OCPs is the reduction of menstrual flow and irregularities. OCPs may be contraindicated for some women and have a number of potential side effects. NSAIDs are the drug of choice. However, if a woman is taking an NSAID she should avoid taking aspirin as well.

A woman has chosen the calendar method of conception control. During the assessment process, it is most important that the nurse: a. Obtain a history of menstrual cycle lengths for the past 6 to 12 months b. Determine the clients weight gain and loss pattern for the previous year c. Examine skin pigmentation and hair texture for hormonal changes d. Explore the clients previous experiences with conception control

Obtain a history of menstrual cycle lengths for the past 6 to 12 months The calendar method of conception control is based on the number of days in each cycle, counting from the first day of menses. The fertile period is determined after the lengths of menstrual cycles have been accurately recorded for 6 months. Weight gain or loss may be partly related to hormonal fluctuations, but it has no bearing on use of the calendar method. Integumentary changes may be related to hormonal changes, but they are not indicators for use of the calendar method. Exploring previous experiences with conception control may demonstrate client understanding and compliancy, but it is not the most important aspect to assess for discussion of the calendar method.

With regard to dysmenorrhea, nurses should be aware that: a.It is more common in older women b. It is more common in leaner women who exercise strenuously c. Symptoms can begin at any point in the ovulatory cycle d. Pain usually occurs in the suprapubic area or lower abdomen

Pain usually occurs in the suprapubic area or lower abdomen Pain is described as sharp and cramping or sometimes as a dull ache. It may radiate to the lower back or upper thighs. Dysmenorrhea is more common in younger women ages 17 to 24. Dysmenorrhea is more common in women who smoke and who are obese. Symptoms begin with menstruation or sometimes a few hours before the onset of flow.

Which statement concerning cyclic perimenstrual pain and discomfort (CPPD) is accurate? a. Premenstrual dysphoric disorder (PDD) is a milder form of premenstrual syndrome and more common in younger women. b. Secondary dysmenorrhea is more intense and medically significant than primary dysmenorrhea. c. Premenstrual syndrome is a complex, poorly understood condition that may include any of a hundred symptoms. d. The causes of premenstrual syndrome (PMS) have been well established.

Premenstrual syndrome is a complex, poorly understood condition that may include any of a hundred symptoms. PMS may manifest itself with one or more of a hundred or so physical and psychologic symptoms. PDD is a more severe variant of PMS. Secondary dysmenorrhea is characterized by more muted pain than that seen in primary dysmenorrhea; the medical treatment is much the same. The cause of PMS is unknown. It may in fact be a collection of different problems.

There is little consensus on the management of premenstrual dysphoric disorder (PMDD). However, nurses can advise women on several self-help modalities that often result in symptom improvement. The nurse knows that health teaching has been effective when the client reports that she has adopted a number of lifestyle changes including: a. Regular exercise b. Improved nutrition c. A daily glass of wine d. Smoking cessation e. Oil of evening primrose

Regular exercise Improved nutrition Smoking cessation Oil of evening primrose Regular exercise, improved nutrition, smoking cessation, and oil of evening primrose are accurate modalities that may provide significant symptom relief in 1 to 2 months. If there is no improvement after these changes have been made, the client may need to begin pharmacologic therapy. Women should decrease their alcohol and caffeinated beverage consumption if they suffer from PMDD.

The volume of amniotic fluid is an important factor in assessing fetal well-being. Oligohydramnios (an amniotic fluid volume of less than 300 ml) is associated with what kind of fetal anomalies? a. Renal b. Cardiac c. Gastrointestinal d. Neurologic

Renal An amniotic fluid volume of less than 300 ml (oligohydramnios) is associated with fetal renal anomalies. The amniotic fluid volume has no bearing on the fetal cardiovascular system. Gastrointestinal anomalies are associated with hydramnios, or an amniotic fluid volume greater than 2 L. The amniotic fluid volume has no bearing on the fetal neurologic system.

Postcoital contraception with Ovral: a. Requires that the first dose be taken within 120 hours of unprotected intercourse b. Requires that the woman take second and third doses at 24 and 36 hours after the first dose c. Has an effectiveness rate in preventing pregnancy of approximately 50% d. Is commonly associated with the side effect of menorrhagia

Requires that the first dose be taken within 120 hours of unprotected intercourse Emergency contraception is used within 120 hours of unprotected intercourse to prevent pregnancy. The first dose of an emergency contraception should be taken within 120 hours after coitus. Postcoital use of Ovral is 74% to 90% effective at preventing pregnancy. The common side effect of postcoital contraception is nausea.

The two primary areas of risk for sexually transmitted infections (STIs) are: a. Sexual orientation and socioeconomic status b. Age and educational level c. Large number of sexual partners and race d. Risky sexual behaviors and inadequate preventive health behaviors

Risky sexual behaviors and inadequate preventive health behaviors Risky sexual behaviors and inadequate preventive health behaviors put a person at risk for acquiring or transmitting an STI. Although low socioeconomic status may be a factor in avoiding purchasing barrier protection, sexual orientation does not put one at higher risk. Younger individuals with less education may not be aware of proper prevention techniques; however, these are not the primary areas for STIs. Having a large number of sexual partners is certainly a risk-taking behavior, but race does not increase the risk for STIs.

It is important for the nurse to understand that the placenta: a. Produces nutrients for fetal nutrition b. Secretes both estrogen and progesterone c. Forms a protective, impenetrable barrier to microorganisms such as bacteria and viruses d. Excretes prolactin and insulin

Secretes both estrogen and progesterone As one of its early functions, the placenta acts as an endocrine gland, producing four hormones necessary to maintain the pregnancy and support the embryo and fetus: human chorionic gonadotropin (hCG), human placental lactogen (hPL), estrogen, and progesterone. The placenta does not produce nutrients. It functions as a means of metabolic exchange between the maternal and fetal blood supplies. Many bacteria and viruses can cross the placental membrane. One of the early functions of the placenta is as an endocrine gland that produces four hormones (hCG, hPL, estrogen, and progesterone) necessary to maintain the pregnancy and support the embryo and fetus.

While interviewing a 31-year-old woman before her routine gynecologic examination, the nurse collects data about the clients recent menstrual cycles. The nurse should collect additional information with which statement? a. The woman says her menstrual flow lasts 5 to 6 days. b. She describes her flow as very heavy. c. She reports that she has had a small amount of spotting midway between her periods for the past 2 months. d. She says the length of her menstrual cycle varies from 26 to 29 days.

She describes her flow as very heavy. Menorrhagia is defined as excessive menstrual bleeding, either in duration or in amount. Heavy bleeding can have many causes. The amount of bleeding and its effect on daily activities should be evaluated. A menstrual flow that lasts 5 to 6 days is a normal finding. Mittlestaining, a small amount of bleeding or spotting that occurs at the time of ovulation (14 days before onset of the next menses), is considered normal. During her reproductive years a woman may have physiologic variations in her menstrual cycle. Variations in the length of a menstrual cycle are considered normal.

Alternatives to hormonal therapy for menopausal symptoms include (choose all that apply): a. Soy b. Vitamin C c. Vitamin K d. Vitamin E e. Vitamin A

Soy Vitamin A Both soy and vitamin E have been reported to help alleviate menopausal symptoms. Vitamins C, K, and A have no apparent effect on menopausal symptoms.

Which contraceptive method has a failure rate of less than 25%? a. Standard days b. Periodic abstinence c. Postovulation d. Coitus interruptus

Standard days The standard days variation on the calendar method has a failure rate of 12%. The periodic abstinence method has a failure rate of 25% or higher. The standard days variation on the calendar method has a failure rate of 12%. The postovulation method has a failure rate of 25% or higher. The coitus interruptus method has a failure rate of 27% or higher.

Your client is undergoing treatment for ovarian cancer. Please identify which common nutritional problems are related to gynecologic cancers and the treatment thereof. a. Stomatitis b. Constipation c. Anorexia d. Diarrhea e. Nausea and vomiting

Stomatitis Constipation Anorexia Diarrhea Nausea and vomiting Stomatitis, constipation, anorexia, diarrhea, and nausea and vomiting are all possible nutritional complications related to gynecologic cancers and their treatment. The nurse must assess accordingly and adapt the clients plan of care. To ensure recovery these women should consume a diet high in iron and protein, drink plenty of fluids, and eat foods high in vitamins C, B and K.

In helping a client manage premenstrual syndrome (PMS), the nurse should: a. Recommend a diet with more body-building and energy food, such as red meat and sugar b. Suggest herbal therapies, as well as yoga and massage c. Tell the client to push for medications from the physician as soon as symptoms occur so as to lessen their severity d. Discourage the use of diuretics

Suggest herbal therapies, as well as yoga and massage Herbal therapies, yoga, and massage have been reported to have a beneficial effect on PMS. Limiting red meat, refined sugar, caffeinated beverages, and alcohol improves the diet and may mitigate symptoms. Medication usually is begun only if lifestyle changes fail to provide significant relief. Natural diuretics may help reduce fluid retention.

A woman will be taking oral contraceptives using a 28-day pack. The nurse should advise this woman to protect against pregnancy by: a. Limiting sexual contact for one cycle after starting the pill b. Using condoms and foam instead of the pill for as long as she takes an antibiotic c. Taking one pill at the same time every day d. Throwing away the pack and using a backup method if she misses two pills during week 1 of her cycle

Taking one pill at the same time every day To maintain adequate hormone levels for contraception and to enhance compliance, clients should take oral contraceptives at the same time each day. If contraceptives are to be started at any time other than during normal menses or within 3 weeks after birth or abortion, another method of contraception should be used through the first week to prevent the risk of pregnancy. Taken exactly as directed, oral contraceptives prevent ovulation, and pregnancy cannot occur. No strong pharmacokinetic evidence indicates a link between the use of broad-spectrum antibiotics and altered hormonal levels in oral contraceptive users. If the client misses two pills during week 1, she should take two pills a day for 2 days and finish the package and use a backup method the next 7 consecutive days.

With regard to the noncontraceptive medical effects of combination oral contraceptives (COCs), nurses should be aware that: a. COCs can cause toxic shock syndrome if the prescription is wrong b. Hormonal withdrawal bleeding usually is a bit more profuse than in normal menstruation and lasts a week c. COCs increase the risk of endometrial and ovarian cancer d. The effectiveness of COCs can be altered by some over-the-counter medications and herbal supplements

The effectiveness of COCs can be altered by some over-the-counter medications and herbal supplements The effectiveness of COCs can be altered by some over-the-counter medications and herbal supplements. Toxic shock syndrome can occur in some diaphragm users, but it is not a consequence of taking oral contraceptive pills. Hormonal withdrawal bleeding usually is lighter than in normal menstruation and lasts a couple of days. Oral contraceptive pills offer protection against the risk of endometrial and ovarian cancers.

With regard to planning treatment for a pregnant woman with breast cancer, which statement about timing or type of treatment is correct? a. The fetus is most at risk during the first trimester. b. The fetus is most at risk during the second trimester. c. The fetus is most at risk during the third trimester. d. Surgery is more risky than chemotherapy in the first trimester.

The fetus is most at risk during the first trimester. The first trimester is the most vulnerable period for the growing fetus. Women may be faced with making a decision about terminating the pregnancy, depending on the stage and extent of the disease. For advanced disease in the second trimester, alkylating agents, 5-fluorouracil (5-FU), and vincristine are relatively safe for the fetus. For advanced disease in the third trimester, alkylating agents, 5-FU, and vincristine are relatively safe for the fetus. Surgery is less risky than chemotherapy in the first trimester.

The nurse should be aware that a pessary would be most effective in the treatment of what disorder? a. Cystocele b. Uterine prolapse c. Rectocele d. Stress urinary incontinence

Uterine prolapse A fitted pessary may be inserted into the vagina to support the uterus and hold it in the correct position. A pessary is not used for the client with a cystocele. A rectocele cannot be corrected by the use of a pessary. It is unlikely that a pessary is the most effective treatment for stress incontinence.

Sally comes in for her first prenatal examination. This is her first child. She asks you (the nurse), How does my baby get air inside my uterus? The correct response is: a. The babys lungs work in utero to exchange oxygen and carbon dioxide. b. The baby absorbs oxygen from your blood system. c. The placenta provides oxygen to the baby and excretes carbon dioxide into your bloodstream. d. The placenta delivers oxygen-rich blood through the umbilical artery to the babys abdomen.

The placenta provides oxygen to the baby and excretes carbon dioxide into your bloodstream. The placenta functions by supplying oxygen and excreting carbon dioxide into the maternal bloodstream. The fetal lungs do not function for respiratory gas exchange in utero. The baby does not simply absorb oxygen from a womans blood system. Blood and gas transport occur through the placenta. The placenta delivers oxygen-rich blood through the umbilical vein, not artery.

An unmarried young woman describes her sex life as active and involving many partners. She wants a contraceptive method that is reliable and does not interfere with sex. She requests an intrauterine device (IUD). The nurses most appropriate response is: a. The IUD does not interfere with sex. b. The risk of pelvic inflammatory disease will be higher for you. c. The IUD will protect you from sexually transmitted infections. d. Pregnancy rates are high with the IUDs.

The risk of pelvic inflammatory disease will be higher for you. Disadvantages of IUDs include an increased risk of pelvic inflammatory disease (PID) in the first 20 days after insertion, as well as the risks of bacterial vaginosis and uterine perforation. The IUD offers no protection against sexually transmitted infections (STIs) or the human immunodeficiency virus (HIV). Because this woman has multiple sex partners, she is at higher risk of developing an STI. The IUD does not protect against infection, as does a barrier method. Stating that an IUD does not interfere with sex may be correct, it is not the most appropriate response. The IUD offers no protection from STIs. The typical failure rate of the IUD ranges from 0.8% to 2%.

A married couple is discussing alternatives for pregnancy prevention and has asked about fertility awareness methods (FAMs). The nurses most appropriate reply is: a. Theyre not very effective, and its very likely youll get pregnant. b. They can be effective for many couples, but they require motivation. c. These methods have a few advantages and several health risks. d. You would be much safer going on the pill and not having to worry.

They can be effective for many couples, but they require motivation. FAMs are effective with proper vigilance about ovulatory changes in the body and with adherence to coitus intervals. Fertility awareness methods are effective if used correctly by a woman with a regular menstrual cycle. The typical failure rate for all FAMs is 25% during the first year of use. FAMs have no associated health risks. The use of birth control has associated health risks. In addition, taking a pill daily requires compliance on the clients part.

Which statement is the most complete and accurate description of medical abortions? a. They are performed only for maternal health. b. They can be achieved through surgical procedures or with drugs. c. They are mostly performed in the second trimester. d. They can be either elective or therapeutic.

They can be either elective or therapeutic. Abortions can be either elective (the womans choice) or therapeutic (for reasons of maternal or fetal health). Abortions might be performed for maternal health or because of the womans choice. Medical abortions are performed through the use of medications (rather than surgical procedures). Medical abortions are usually performed in the first trimester.

The Centers for Disease Control and Prevention (CDC) recommends that human papillomavirus (HPV) be treated with client-applied: a. Miconazole ointment b. Topical podofilox 0.5% solution or gel c. Penicillin given intramuscularly for two doses d. Metronidazole by mouth

Topical podofilox 0.5% solution or gel Available treatments are imiquimod, podophyllin, and podofilox. Miconazole ointment is used to treat athletes foot. Penicillin IM is used to treat syphilis. Metronidazole is used to treat bacterial vaginosis.

A saline wet smear (vaginal secretions mixed with normal saline on a glass slide) is the test for: a. Bacterial vaginosis b. Candidiasis c. Yeast infection d. Trichomoniasis

Trichomoniasis The presence of many white blood cell protozoa is a positive finding for trichomoniasis. A normal saline test is used to test for bacterial vaginosis. A potassium hydroxide preparation is used to test for candidiasis. Yeast infection is the common name for candidiasis, for which the test is a potassium hydroxide preparation.

Which statements might the nurse appropriately include when teaching a client about calcium intake for osteoporosis? Choose all that apply. a. You should try to increase your protein intake when you are taking calcium. b. It is best to take calcium in one large dose. c. Tums are the most soluble form of calcium. d. You should take calcium with vitamin D because the vitamin D helps your body absorb calcium better. e. Its okay to take calcium if you have had a history of kidney stones.

Tums are the most soluble form of calcium. You should take calcium with vitamin D because the vitamin D helps your body absorb calcium better. Teaching the client to take calcium with vitamin D or to take calcium with a history of kidney stones is accurate. Excessive protein should be avoided. Calcium is best taken in divided doses so as to increase absorption. Calcium should be taken with vitamin D to increase absorption. Calcium is contraindicated in women with a history of kidney stones.

A woman who has just undergone a first-trimester abortion will be using oral contraceptives. To protect against pregnancy, she should be advised to: a. Avoid sexual contact for at least 10 days after starting the pill b. Use condoms and foam for the first few weeks as backup c. Use another method of contraception for 1 week after starting the pill d. Begin sexual relations once vaginal bleeding has ended

Use another method of contraception for 1 week after starting the pill If contraceptives are to be started within 3 weeks after an abortion, another method of contraception should be used throughout the first week to avoid the risk of pregnancy. If contraceptives are to be started within 3 weeks after an abortion, another method of contraception should be used throughout the first week to avoid the risk of pregnancy. Additional forms of contraception should be used for 1 week after starting on oral contraceptives. If contraceptives are to be started within 3 weeks after an abortion, another method of contraception should be used throughout the first week to avoid the risk of pregnancy.

When a nurse is counseling a woman for primary dysmenorrhea, which nonpharmacologic intervention might be recommended? a. Increasing the intake of red meat and simple carbohydrates b. Reducing the intake of diuretic foods, such as peaches and asparagus c. Temporarily substituting physical activity for a sedentary lifestyle d. Using a heating pad on the abdomen to relieve cramping

Using a heating pad on the abdomen to relieve cramping Heat minimizes cramping by increasing vasodilation and muscle relaxation and minimizing uterine ischemia. Dietary changes such as eating less red meat may be recommended for women experiencing dysmenorrhea. Increasing the intake of diuretics, including natural diuretics such as asparagus, cranberry juice, peaches, parsley, and watermelon may help ease the symptoms associated with dysmenorrhea. Exercise has been found to help relieve menstrual discomfort through increased vasodilation and subsequent decreased ischemia.

The nurse working with a client who has infertility concerns should be aware that leuprolide acetate (Lupron), a gonadotropin-releasing hormone (GnRH) agonist, may be prescribed for an infertile woman to treat: a. Anovulatory cycles b. Uterine fibroids c. Polycystic ovary disease (PCOD) d. Luteal phase inadequacy

Uterine fibroids Leuprolide acetate is used to treat endometriosis and uterine fibroids. Anovulatory cycles are treated with Clomid, Serophene, Pergonal, or Profasi, all of which stimulate ovulation induction. Metrodin is used to treat PCOD. Progesterone is used to treat luteal phase inadequacy.

Although reported in small numbers, toxic shock syndrome can occur with the use of a diaphragm. If a client is interested in this form of conception control, the nurse should teach the woman how to reduce her risk of TSS. The nurse might say: a. You should always remove your diaphragm 6 to 8 hours after intercourse. Dont use the diaphragm during menses, and watch for danger signs of TSS, including a sudden onset of fever over 38.4 C, hypotension, and a rash. b. You should remove your diaphragm right after intercourse to prevent TSS. c. Its okay to use your diaphragm during your menstrual cycle. Just be sure to wash it thoroughly first to prevent TSS. d. Make sure you dont leave your diaphragm in for longer than 24 hours, or you may get TSS.

You should always remove your diaphragm 6 to 8 hours after intercourse. Dont use the diaphragm during menses, and watch for danger signs of TSS, including a sudden onset of fever over 38.4 C, hypotension, and a rash. The nurse should instruct the client on proper use and removal of the diaphragm, and include the danger signs of TSS. The diaphragm must remain against the cervix for 6 to 8 hours to prevent pregnancy, but it should not remain in place longer than 8 hours to avoid the risk of TSS. The diaphragm should not be used during menses. The diaphragm needs to remain against the cervix for 6 to 8 hours to prevent pregnancy, but it should not remain in place longer than 8 hours to avoid the risk of TSS.

A woman who is 8 months pregnant asks the nurse, Does my baby have any antibodies to fight infection? The most appropriate response by the nurse is: a. Your baby has all the immunoglobulins necessary: IgG, IgM, and IgA. b. Your baby wont receive any antibodies until he is born and you breastfeed him. c. Your baby does not have any antibodies to fight infection. d. Your baby has IgG and IgM immunoglobulins.

Your baby has IgG and IgM immunoglobulins. During the third trimester the only immunoglobulin that crosses the placenta, IgG, provides passive acquired immunity to specific bacterial toxins. The fetus produces IgM by the end of the first trimester. During the third trimester the only immunoglobulin that crosses the placenta, IgG, provides passive acquired immunity to specific bacterial toxins. The fetus produces IgM by the end of the first trimester. IgA immunoglobulins are not produced by the baby. By the third trimester the fetus has IgG and IgM. Breastfeeding supplies the baby with IgA immunoglobulins. By the third trimester, the fetus has IgG and IgM immunoglobulins to fight infection.

A woman asks the nurse, What protects my babys umbilical cord from being squashed while the babys inside of me? The nurses best response is: a. Your babys umbilical cord is surrounded by connective tissue called Wharton jelly, which prevents compression of the blood vessels and ensures continued nourishment of your baby. b. Your babys umbilical floats around in blood anyway. c. You dont need to be worrying about things like that. d. The umbilical cord is a group of blood vessels that are very well protected by the placenta.

Your babys umbilical cord is surrounded by connective tissue called Wharton jelly, which prevents compression of the blood vessels and ensures continued nourishment of your baby. Explaining the structure and function of the umbilical cord is the most appropriate response. The umbilical cord does not float around in blood. Telling the client not to worry negates her need for teaching and discounts her feelings. The placenta does not protect the umbilical cord. The cord is protected by the surrounding Wharton jelly.


Ensembles d'études connexes

Lab Simulation 5-2: Test the Efficiency of your Network Network +

View Set

Биохимия 1 (50 тестов укр )

View Set

Nat. Geo. 13 Colonies: Section 1

View Set

Developmental Psychology Chapter 12: Family

View Set

Extensions of Mendelian Inheritance

View Set

Ch.2 Strategic Planning (LearnSmart)

View Set

ATI Video Case Study: Urinary Tract Infections.

View Set

Pharmacology PrepU Chapter 30 Skeletal Muscle, Bone, and Joint Disorder Drugs

View Set